SlideShare a Scribd company logo
1 of 69
Download to read offline
www.VNMATH.com




    PHƯƠNG TRÌNH HÀM - K THU T GI I VÀ
         M T S V N Đ LIÊN QUAN
        Tr n Minh Hi n - GV trư ng THPT chuyên Quang Trung, Bình Phư c

                                    Ngày 15 tháng 6 năm 2011



M cl c
M cl c                                                                                                  1

1 Phương pháp th bi n                                                                                   2

2 Phương trình hàm Cauchy                                                                              12

3 Phương pháp quy n p                                                                                  19

4 Khai thác tính ch t đơn ánh, toàn ánh, song ánh, ch n l c a hàm s                                    24

5 Khai thác tính đơn đi u c a hàm s                                                                    34

6 Khai thác tính ch t đi m b t đ ng c a hàm s                                                          40

7 Phương pháp đưa v phương trình sai phân                                                              44

8 Phương pháp s d ng tính liên t c c a hàm s                                                           46

9    ng d ng phương trình hàm cơ b n                                                                   53

10 B t đ ng th c hàm                                                                                   60

11 Hàm tu n hoàn                                                                                       65

12 M t s chuyên đ phương trình hàm                                                             66
   12.1 Phương trình hàm gi i nh tính giá tr hàm s theo hai cách khác nhau . . . . . . . . . . 66

13 Gi i phương trình hàm b ng cách thêm bi n                                                           68

14 LUY N T P PHƯƠNG TRÌNH HÀM                                                                           69
   14.1 Phương pháp th bi n . . . . . . . . . . . . . . . . . . . . . . . . . . . . . . . . . . . . . 69
   14.2 B t đ ng th c hàm . . . . . . . . . . . . . . . . . . . . . . . . . . . . . . . . . . . . . . . 69


                                                    1
www.VNMATH.com                  1 PHƯƠNG PHÁP TH BI N


1      Phương pháp th bi n
    Phương pháp th bi n có l là phương pháp đư c s d ng nhi u nh t khi gi i phương trình hàm. Ta
có th :

    • Ho c cho các bi n x, y, . . . nh n các giá tr b ng s . Thư ng các giá tr đ c bi t là 0, ±1, ±2, . . .

    • Ho c th các bi n b ng các bi u th c đ làm xu t hi n các h ng s ho c các bi u th c c n thi t.
      Ch ng h n, n u trong phương trình hàm có m t f (x + y) mà mu n có f (0) thì ta th y b i −x,
      mu n có f (x) thì cho y = 0, mu n có f (nx) thì th y b i (n − 1)x.

Ví d 1.1. (Áo 199?) Tìm t t c các hàm s f : R → R th a mãn đi u ki n

                                   x2 f (x) + f (1 − x) = 2x − x4 , ∀x ∈ R.

Gi i

Thay x b i 1 − x ta đư c

                         (1 − x)2 f (1 − x) + f (x) = 2(1 − x) − (1 − x)4 , ∀x ∈ R.

Nhu v y ta có h            8
                           <
                            x2 f (x) + f (1 − x) = 2x − x4
                           :f (x) + (1 − x)2 f (1 − x) = 2(1 − x) − (1 − x)4
                                                                                 .

Ta có D = (x2 − x − 1) (x2 − x + 1) và Dx = (1 − x2 ) (x2 − x − 1) (x2 − x + 1). V y D.f (x) = Dx , ∀x ∈
R. T đó ta có nghi m c a bài toán là
                               8
                               >
                               >
                               <
                                1 − x2         : x = a, x = b,
                       f (x) = >c ∈ R          : x = a,        (c là h ng s tùy ý),
                               >
                               :
                                2a − a4 − a2 c : x = b,

v i a, b là nghi m c a phương trình x2 − x − 1 = 0.

    Nh n xét: Bài toán trên đư c dùng m t l n n a trong kỳ thi VMO 2000, b ng B.

Ví d 1.2. Tìm t t c các hàm s f : R → R th a mãn đi u ki n

                               f (x + y) + f (x − y) = 2f (x) cos y, ∀x, y ∈ R

    Hint: 1. Th y → π  2
2. Th y → y + π ho c th x = π
                2                 2
3. Th x → 0
Đáp s : f (x) = a cos x + b sin x(a, b ∈ R)

Ví d 1.3. f : R → R th a mãn đi u ki n f (xy + x + y) = f (xy) + f (x) + f (y), x, y ∈ R. Ch ng minh
r ng:
                              f (x + y) = f (x) + f (y), ∀x, y ∈ R.


GV: Tr n Minh Hi n . . . . . . PTH b i dư ng h c sinh gi i . . . . . . Trư ng THPT chuyên Quang Trung
www.VNMATH.com                               1 PHƯƠNG PHÁP TH BI N


      Hint:
1.   Tính f (0)
2.   Th y = −1, ch ng minh f là hàm l
3.   Th y = 1 ⇒ f (2x + 1) = 2f (x) + 1
4.   Tính f (2(u + v + uv) + 1) theo (3) và theo gi thi t đ suy ra f (2uv + u) = 2f (uv) + f (u)
5.              1
     Cho v = − 2 , u → x và u → y, 2uv → x đ suy ra đi u ph i ch ng minh
                   2

Ví d 1.4. Tìm t t c các hàm s f : R → R đ ng th i th a mãn các đi u ki n sau:
                                                                      
                                                         1
                                           f (x) = xf       , ∀x = 0
                                                         x
                      f (x) + f (y) = 1 + f (x + y), ∀x, y ∈ R, (x, y) = (0, 0); x + y = 0
    Hint:
1. Tính f (0), f (−1)          €     Š €         Š
2. Tính a + 1 v i a = f (1) = f x+1 = f x + 1 x+1 theo c hai đi u ki n.
                                 x+1
                                               1

Đáp s : f (x) = x + 1

     Nh n xét: Th thu t này áp d ng cho m t l p các bài toán g n tuy n tính
                                                                               1
Ví d 1.5. Tìm t t c các hàm s f : R+ → R th a f (1) =                          2
                                                                                   và
                                                   ‚       Œ                  
                                                       3                3
                               f (xy) = f (x)f                 + f (y)f   , ∀x, y ∈ R+
                                                       y                x
    Hint:
1. Tính f (3)
             3
2. Th y → x
                  1
Đáp s : f (x) =   2

Ví d 1.6. Tìm t t c các hàm s f : R∗ → R th a mãn đi u ki n:
                                                                  
                                                   1
                                        f (x) + 2f   = 3x, ∀x ∈ R∗
                                                   x
                   1
  Hint: Th x → x
                2
Đáp s : f (x) = x − x

Ví d 1.7. Tìm t t c các hàm s f : R{0, 1} → R th a mãn đi u ki n:
                                                              
                                                   x−1
                                   f (x) + f           = 2x, ∀x, ∈ R{0, 1}
                                                    x
  Hint:
                   −1
Th x → x−1 , x → x−1
           x
                     1       x−1
Đáp s : f (x) = x + 1−x −     x
Luy n t p:

     2. Tìm t t c các hàm s f : Q+ → Q+ th a mãn đi u ki n:

                         f (x + 1) = f (x) + 1, ∀x ∈ Q+ và f (x3 ) = f 3 (x), ∀x ∈ Q+

GV: Tr n Minh Hi n . . . . . . PTH b i dư ng h c sinh gi i . . . . . . Trư ng THPT chuyên Quang Trung
www.VNMATH.com                1 PHƯƠNG PHÁP TH BI N


Hint:
1. Quy n p f (x + n) = f (x) + n,‹ ∈ Q+ , ∀n ∈ N
                     
                                  ∀x
                                           3
         p                       p
2. V i   q
             ∈ Q+ , tính f       q
                                     + q2            theo hai cách.
Đáp s : f (x) = x, ∀x ∈ Q+
Ví d 1.8. (VMO 2002). Hãy tìm t t c các hàm s f (x) xác đ nh trên t p s th c R và th a mãn
h th c                             €         Š
                  f (y − f (x)) = f x2002 − y − 2001.y.f (x), ∀x, y ∈ R.                (1)

Gi i

a) Th y = f (x) vào (1) ta đư c
                                                        €             Š
                                       f (0) = f x2002 − f (x) − 2002. (f (x))2 , ∀x ∈ R.                    (2)

b) L i thay y = x2002 vào (1) thì
                                        €                       Š
                                      f x2002 − f (x) = f (0) − 2001.x2002 .f (x), ∀x ∈ R.                   (3)

L y (2) c ng v i (3) ta đư c                                €             Š
                                                     f (x) f (x) + x2002 = 0, ∀x ∈ R.
T đây suy ra v i m i giá tr x ∈ R thì ta có ho c là f (x) = 0 ho c là f (x) = −x2002 . Ta s ch ra r ng
đ th a mãn yêu c u bài toán thì b t bu c ph i có đ ng nh t

                                       f (x) ≡ 0, ∀x ∈ R ho c f (x) ≡ −x2002 , ∀x ∈ R.

Th t v y, vì f (0) = 0 trong c hai hàm s trên, nên không m t tính t ng quát ta có th gi s t n t i
a = 0 sao cho f (a) = 0, và t n t i b  0 sao cho f (b) = −b2002 (vì ch c n thay x = 0 vào quan h (1) ta
nh n đư c hàm f là hàm ch n). Khi đó th x = a và y = −b vào (1) ta đư c
                                                                      €        Š
                                                            f (−b) = f a2002 + b .

V y ta nh n đư c dãy quan h sau

                      0 = −b2002
                        = f (b)
                        = f (−b)
                                 €               Š
                        = f a2002 + b
                             
                                 0(mâu thu n vì 0 = 0)
                        =                     2002                            2002            .
                                 − (a2002 + b)     (mâu thu n vì − (a2002 + b)      −b2002 )

B ng cách th l i quan h hàm ban đ u ta k t lu n ch có hàm s f (x) ≡ 0, ∀x ∈ R th a mãn yêu c u
bài toán.
Ví d 1.9. (Hàn Qu c 2003) Tìm t t c các hàm s f : R → R th a mãn

                                 f (x − f (y)) = f (x) + xf (y) + f (f (y)) ,           ∀x, y ∈ R.           (4)

GV: Tr n Minh Hi n . . . . . . PTH b i dư ng h c sinh gi i . . . . . . Trư ng THPT chuyên Quang Trung
www.VNMATH.com                     1 PHƯƠNG PHÁP TH BI N


Gi i

Nh n th y hàm f (x) ≡ 0 th a mãn yêu c u bài toán. Xét trư ng h p f (x) ≡ 0.
a) Th x = f (y) vào (4) ta đư c
                                                                    x2 f (0)
                                 f (0) = 2f (x) + x2 → f (x) = −      +      ,
                                                                    2    2
   hay
                                                         f 2 (x) f (0)
                                         f (f (x)) = −          +      .
                                                            1      2
b) Th x = f (z), v i z là m t s thu c R thì ta đư c
                             f (f (z) − f (y)) = f (f (z)) + f (z)f (y) + f (f (y)) .
   V i lưu ý là
                                       f 2 (y) f (0)                  f 2 (z) f (0)
                         f (f (y)) = −        +      và f (f (z)) = −        +      ,
                                          2      2                       2      2
   thay vào quan h hàm       trên ta đư c
                                                      (f (z) − f (y))2
                                f (f (z) − f (y)) = −                  + f (0).                     (5)
                                                             2

c) Ti p theo ta ch ng t t p {f (x) − f (y)|x, y ∈ R} = R. Do f (x) ≡ 0 nên t n t i m t giá tr y0 sao
   cho f (y0 ) = a = 0. Khi đó t quan h (4) ta có
                        f (x − a) = f (x) + xa + f (a) → f (x − a) − f (x) = ax + f a.
   Vì v ph i là hàm b c nh t c a X nên xa + f a có t p giá tr là toàn b R. Do đó hi u f (x − a) − f (x)
   cũng có t p giá tr là toàn b R, khi x ∈ R. Mà
                          {f (x) − f (y)|x, y ∈ R} ⊃ {f (x − a) − f (x)|x ∈ R} = R,
   do đó {f (x) − f (y)|x, y ∈ R} = R. V y t quan h (5) ta thu đư c
                                                     x2
                                         f (x) = −      + f (0), ∀x ∈ R.
                                                     2
   M t khác ta l i có
                                             x2
                                    f (x) = − + f (0), ∀x ∈ T (f )
                                             2
                                              x2
   nên f (0) = 0. Th l i th y hàm s f (x) = − , ∀x ∈ R th a mãn quan h hàm.
                                              2
                                                   x2
K t lu n: Có hai hàm s th a mãn là f (x) = −          , ∀x ∈ R ho c f (x) ≡ 0.
                                                   2
  Nh n xét: Bài toán trên l y ý tư ng t bài thi IMO 1996: Tìm t t c các hàm s f : R → R th a
mãn
                     f (x − f (y)) = f (f (y)) + xf (y) + f (x) − 1, ∀x, y ∈ R.
                     x2
Đáp s là f (x) = −      + 1, ∀x ∈ R.
                     2
GV: Tr n Minh Hi n . . . . . . PTH b i dư ng h c sinh gi i . . . . . . Trư ng THPT chuyên Quang Trung
www.VNMATH.com              1 PHƯƠNG PHÁP TH BI N


Ví d 1.10. (Iran 1999) Xác đ nh các hàm s f : R → R th a mãn
                                                     €     Š
                             f (f (x) + y) = f x2 − y + 4yf (x), ∀x, y ∈ R.

Gi i

a) Th y = x2 ta đư c                   €             Š
                                     f f (x) + x2 = f (0) + 4x2 f (x), ∀x ∈ R.

b) Th y = −f (x) ta đư c                         €        Š
                                     f (0) = f f (x) + x2 − 4 (f (x))2 , ∀x ∈ R.

C ng hai phương trình trên ta đư c
                                                 €         Š
                                           4f (x) f (x) − x2 = 0, ∀x ∈ R.

T đây ta th y v i m i x ∈ R thì ho c là f (x) ≡ 0 ho c là f (x) = −x2 . Ta ch ng minh n u hàm
f th a mãn yêu c u bài toán thì f ph i đ ng nh t v i hai hàm s trên. Nh n th y f (0) = 0, t đó
thay x = 0 ta đư c f (y) = f (−y), ∀y ∈ R, hay f là hàm ch n. Gi s t n t i a = 0, b = 0 sao cho
f (a) = 0, f (b) = −b2 , khi đó thay x = a, y = −b ta đư c

                                     f (−b) = f (a2 + b) → f (b) = f (a2 + b).

T đó ta có quan h sau

                        0 = −b2
                          = f (b)
                          = f (−b)
                                 €          Š
                         = f a2 + b
                             
                                 0(mâu thu n vì 0 = 0)
                         =                 2                        2        .
                                 − (a2 + b) (mâu thu n vì − (a2 + b)  −b2 )

Do đó x y ra đi u mâu thu n. Th l i th y hàm s f (x) ≡ 0 th a mãn yêu c u.

    Nh n xét:
1. Rõ ràng bài toán VMO 2002 có ý tư ng gi ng bài toán này.
2. Ngoài phép th như trên thì bài toán này ta cũng có th th c hi n nh ng phép th khác như sau:
            1€ 2       Š
a) Th y =     x − f (x) .
            2
b) Th y = 0 đ có f (f (x)) = f (x2 ), sau đó th y = x2 − f (x).

c) Th y = x − f (x) và sau đó là y = x2 − x.
Ví d 1.11. Tìm hàm s f : R → R th a mãn đi u ki n:

                                 f (x − f (y)) = 2f (x) + x + f (y), ∀x, y ∈ R.                   (6)

Gi i

GV: Tr n Minh Hi n . . . . . . PTH b i dư ng h c sinh gi i . . . . . . Trư ng THPT chuyên Quang Trung
www.VNMATH.com                    1 PHƯƠNG PHÁP TH BI N


Nh n th y hàm f (x) ≡ 0 không th a mãn yêu c u. Xét f (x) ≡ 0.
a) Thay x b i f (y) vào (6) ta đư c
                                                                  f (0)
                                           f (f (y)) = −f (y) +         .
                                                                    2
b) L i thay x b i f (x) ta đư c

                            f (f (x) − f (y)) = 2f (f (x)) + f (x) + f (y)
                                                 ‚                  Œ
                                                              f (0)
                                              = 2 −f (x) +            + f (x) + f (y)
                                                                2
                                              = − (f (x) − f (y)) + f (0).

   Tuy nhiên vi c ch ng minh t p {f (x) − f (y)|x, y ∈ R} có t p giá tr là R chưa th c hi n đư c.

c) T đây ta có

                         f (f (x) − 2f (y)) = f ((f (x) − f (y)) − f (y))
                                            = 2f (f (x) − f (y)) + f (x) − f (y) + f (y)
                                            = −2 (f (x) − f (y)) + 2f (0) + f (x)
                                            = − (f (x) − 2f (y)) + 2f (0).

   Ta s ch ng minh t p {f (x) − 2f (y)|x, y ∈ R} b ng v i R. Th t v y t n t i giá tr y0 ∈ R sao cho
   f (y0 ) = a = 0. Khi đó thay y = y0 vào (6) ta có

                                      f (x − a) − 2f (x) = x + a, ∀x ∈ R.

   Mà khi x ∈ R thì x + a có t p giá tr là R. Ch ng t t p {f (x − a) − f (x)|x ∈ R} = R. Mà
   {f (x) − 2f (y)|x, y ∈ R} ⊃ {f (x − a) − f (x)|x ∈ R} nên {f (x) − 2f (y)|x, y ∈ R} = R. Do đó t (c)
   ta k t lu n f (x) = −x + 2f (0), ∀x ∈ R. Thay vào (6) ta đư c f (0) = 0.
K t lu n: Hàm s f (x) = −x, ∀x ∈ R th a mãn yêu c u bài toán.
Ví d 1.12. (Belarus 1995) Tìm t t c các hàm s f : R → R th a mãn

                          f (f (x + y)) = f (x + y) + f (x)f (y) − xy, ∀x, y ∈ R.

Gi i

Rõ ràng f khác h ng s .
a) y = 0 vào đi u ki n bài toán ta đư c

                                      f (f (x)) = (1 + f (0)) f (x), ∀x ∈ R.

b) Trong đ ng th c trên thay x b i x + y thì

                      (1 + f (0)) f (x + y) = f (f (x + y)) = f (x + y) + f (x)f (y) − xy,

   đơn gi n ta đư c
                                       f (0).f (x + y) = f (x)f (y) − xy.                           (7)

GV: Tr n Minh Hi n . . . . . . PTH b i dư ng h c sinh gi i . . . . . . Trư ng THPT chuyên Quang Trung
www.VNMATH.com                     1 PHƯƠNG PHÁP TH BI N


c) Thay y = 1 vào (7) thì
                                        f (0)f (x + 1) = f (x)f (1) − x.

d) L i thay y = −1 và x b i x + 1 vào (7) ta có

                                     f (0).f (x) = f (x + 1).f (−1) + x + 1.

   K t h p hai đ ng th c trên ta đư c
                          €                       Š
                            (f (0))2 − f (1)f (−1) f (x) = (f (0) − f (−1)) x + f (0).

N u (f (0))2 − f (1)f (−1) = 0, thì thay x = 0 vào phương trình cu i cùng ta đư c f (0) = 0, nên theo
(7) thì f (x)f (y) = xy. Khi đó f (x)f (1) = x, ∀x ∈ R, đi u này d n đ n (f (0))2 − f (1)f (−1) = −1, mâu
thu n. V y (f (0))2 −f (1)f (−1) = 0, suy ra f (x) là m t đa th c b c nh t nên có d ng f (x) = ax+b. Thay
vào quan h hàm ban đ u suy ra a = 1, b = 0. V y hàm s th a mãn yêu c u bài toán là f (x) = x, ∀x ∈ R.

   Nh n xét: N u ch u khó tính ta s tính đư c f (0) = 0 b ng cách th các bi n x, y b i hai s 0 và 1.
Ví d 1.13. (VMO 2005) Hãy xác đ nh t t c các hàm s f : R → R th a mãn đi u ki n

                        f (f (x − y)) = f (x)f (y) − f (x) + f (y) − xy, ∀x, y ∈ R.                   (8)

Gi i

a) Th x = y = 0 vào (8) ta đư c
                                              f (f (0)) = (f (0))2 .

b) Th x = y vào (8) và s d ng k t qu trên thì

                                        (f (x))2 = (f (0))2 + x2 , ∀x ∈ R.

   Suy ra (f (x))2 = (f (−x))2 → |f (x)| = |f (−x)| , ∀x ∈ R.
c) Th y = 0 vào (8) đư c

                              f (f (x)) = f (0)f (x) − f (x) + f (0), ∀x ∈ R (∗).

d) Th x = 0, y = −x vào (8) đư c

                                 f (f (x)) = f (0)f (−x) + f (−x) − a, ∀x ∈ R.

   T hai đ ng th c trên ta có

                            f (0) (f (−x) − f (x)) + f (−x) + f (x) = 2f (0), ∀x ∈ R.                 (9)

Gi s t n t i x0 = 0 sao cho f (x0 ) = f (−x0 ), thì th x = x0 vào (9) ta có

                                         f (x0 ) = f (0)
                                       → (f (x0 ))2 = (f (0))2
                                       → (f (0))2 + x2 = (f (0))2 + 02
                                                     0
                                       →x0 = 0 mâu thu n

GV: Tr n Minh Hi n . . . . . . PTH b i dư ng h c sinh gi i . . . . . . Trư ng THPT chuyên Quang Trung
www.VNMATH.com                          1 PHƯƠNG PHÁP TH BI N


V y f (x) = −f (x), ∀x ∈ R, t đi u này k t h p v i (9) ta có

                                      f (0) (f (x) − 1) = 0, ∀x ∈ R.

T đây suy ra f (0) = 0, vì n u ngư c l i thì f (x) = 1, ∀x = 0, trái v i đi u ki n f là hàm l . T đây ta
nh n đư c quan h quen thu c
                                        (f (x))2 = x2 , ∀x ∈ R.
Gi s t n t i x0 ∈ R sao cho f (x0 ) = x0 , khi đó trong (*) ta có

                               x0 = f (x0 ) = −f (f (x0 )) = −f (x0 ) = x0 ,

vô lý. V y ch ng t f (x) = −x, ∀x ∈ R. Th l i th y hàm này th a mãn bài toán.
    Nh n xét: Bài toán trên cho k t qu là hàm ch n f (x) = −x. N u v n gi a nguyên v ph i và đ
nh n đư c hàm l f (x) = x, ta s a l i d ki n trong v trái như trong ví d sau

Ví d 1.14. Tìm t t c các hàm s f : R → R th a mãn đi u ki n

                        f (f (x) − y) = f (x) − f (y) + f (x)f (y) − xy, ∀x, y ∈ R.

Gi i

a) Th y = 0 ta đư c
                               f (f (x)) = f (x) − f (0) + f (0).f (x), ∀x ∈ R.                    (10)

b) Th y = f (x) và s d ng k t qu trên, ta đư c

                       f (0) = f (x) − f (f (x)) + f (x).f (f (x)) − xf (x) (∗)
                            = f (0) − 2f (0).f (x) + (f (x))2 + f (0). (f (x))2 − xf (x),

   hay
                       −2f (0).f (x) + (f (x))2 + f (0). (f (x))2 − xf (x) = 0, ∀x ∈ R.

c) Th x = 0 vào đ ng th c trên ta đư c

                             (f (0))2 − (f (0))2 = 0 → f (0) = 0 ho c f (0) = 1.

d) N u f (0) = 0 thì thay vào (10) ta có f (f (x)) = f (x), ∀x ∈ R, thay k t qu này vào trong (*) ta có
   f (x) = x.
                                                                                             1
e) N u f (0) = 1 thay vào (10) ta có f (f (x)) = 2f (x) − 1, thay vào trong (*) ta có f (x) = x + 1.
                                                                                             2

K t lu n: Thay vào ta th y ch có hàm s f (x) = x, ∀x ∈ R là th a mãn yêu c u.

Ví d 1.15. (AMM,E2176). Tìm t t c các hàm s f : Q → Q th a mãn đi u ki n
                                             ‚         Œ
                                                 x+y           f (x) + f (y)
                            f (2) = 2 và f                 =                 , ∀x = y.
                                                 x−y           f (x) − f (y)

Gi i

GV: Tr n Minh Hi n . . . . . . PTH b i dư ng h c sinh gi i . . . . . . Trư ng THPT chuyên Quang Trung
www.VNMATH.com                    1 PHƯƠNG PHÁP TH BI N


Ta s ch ng minh f (x) = x là nghi m duy nh t c a bài toán d a vào m t chu i các s ki n sau. Trư c
tiên nh n th y f không th là hàm h ng.

a) Tính f (0), f (1). Thay y = 0 ta nh n đư c

                               f (x) + f (0)
                     f (1) =                 → (f (1) − 1) f (x) = f (0) (1 + f (1)) , ∀x ∈ Q.
                               f (x) − f (0)

   Suy ra f (1) = 1, f (0) = 0.

b) Hàm f là hàm l . Thay y = −x ta có

                             0 = f (0) = f (x) + f (−x) → f (−x) = −f (x), ∀x ∈ Q.

c) Thay y = cx, c = 1, x = 0 ta có
                                                                
                                          f (x) + f (cx)    1+c   1 + f (c)
                                                         =f     =           ,
                                          f (x) − f (cx)    1−c   1 − f (c)
                                                                  ‚       Œ
                                              p                       p           f (p)
   suy ra f (cx) = f (c).f (x), l y c = q, x = thì ta đư c f                  =
                                              q                       q           f (q)

Ví d 1.16. Tìm t t c các hàm s f : R → R th a mãn
                                  €           Š
                               f (x − y)2 = (f (x))2 − 2xf (y) + y 2 , ∀x, y ∈ R.

Gi i

Thay x = y = 0 thì (f (0)) = (f (0))2 → f (0) = 0 ho c f (0) = 1.

  1. N u f (0) = 0, thì thay x = y vào đi u ki n ban đ u ta đư c

                       f (0) = (f (x))2 − 2xf (x) + x2 = (f (x) − x)2 → f (x) = x, ∀x ∈ R.

       Nh n th y hàm s này th a mãn.

  2. N u f (0) = 1 thì l i v n thay x = y = 0 ta nh n đư c, v i m i x ∈ R thì ho c là f (x) = x + 1
     ho c f (x) = x − 1. Gi s t n t i giá tr a sao cho f (a) = a − 1. Khi đó thay x = a, y = 0 ta đư c
                                                     €   Š
                                                    f a2 = a2 − 4a + 1.

       Nhưng ta l i có ho c là f (a2 ) = a2 + 1 ho c là f (a2 ) = a2 − 1. Do đó ta ph i có ho c là
                                                                            1
       a2 − 4a + 1 = a2 + 1 ho c a2 − 4a + 1 = a2 − 1, t c a = 0 ho c là a = . Tuy nhiên ki m tra đ u
                                                                            2
       không th a.

V y hàm s th a mãn yêu c u là f (x) = x, ∀x ∈ R ho c là f (x) = x + 1, ∀x ∈ R.

Ví d 1.17. (THTT T9/361) Tìm t t c các hàm s f : R → R th a mãn đi u ki n
                         €            Š      €               Š
                       f x3 − y + 2y 3 (f (x))2 + y 3 = f (x + f (y)) , ∀x, y ∈ R.

GV: Tr n Minh Hi n . . . . . . PTH b i dư ng h c sinh gi i . . . . . . Trư ng THPT chuyên Quang Trung
www.VNMATH.com                     1 PHƯƠNG PHÁP TH BI N


Gi i

a) Thay y = x3 ta có                                         €                    Š   €           Š
                                       f (0) + 2x3 3 (f (x))2 + x6 = f x3 + f (x) , ∀x ∈ R.

b) Thay y = −f (x) ta đư c
                                   €                 Š                   €                    Š
                                  f x3 + f (x) − 2f (x) 3 (f (x))2 + (f (x))2 = f (0), ∀x ∈ R.

T hai đ ng th c trên ta đư c
                                                     €                        Š
                                            2x3 3 (f (x))2 + x6 = 8 (f (x))3 , ∀x ∈ R.

Do đó
                              €                          Š
   0 = 4 (f (x))2 − x3 3 (f (x))2 + x6
         €                                   Š           €                        Š
       = 4 (f (x))3 − 4 (f (x))2 .x3 + (f (x))2 .x3 − x9
                                                                                                            ‚             Œ2       !
         €
                     3
                         Š€
                                        2        3
                                                     €
                                                                     3
                                                                         ŠŠ               €
                                                                                                    3
                                                                                                        Š            x3         15
       = f (x) − x        4 (f (x)) + x                  f (x) + x                    = f (x) − x           2f (x) +           + x6 .
                                                                                                                     4          16
              ‚                    Œ2
                      x3     15
Chú ý r ng 2f (x) +        + x6 = 0 thì x = 0, f (0) = 0. B i v y trong m i trư ng h p ta đ u có
                       4     16
         3
f (x) = x . Th l i th y hàm s này th a mãn bài toán.




GV: Tr n Minh Hi n . . . . . . PTH b i dư ng h c sinh gi i . . . . . . Trư ng THPT chuyên Quang Trung
www.VNMATH.com 2 PHƯƠNG TRÌNH HÀM CAUCHY


2      Phương trình hàm Cauchy
    PHƯƠNG TRÌNH HÀM CÔSI(HÀM TUY N TÍNH) Version 5.0 updated to 24 – 10 – 2008 I.Đ nh
nghĩa: M t hàm s f : R → R g i là tuy n tính n u: f (x + y) = f (x) + f (y), ∀x, y ∈ R (Hàm s tuy n
tính còn đư c g i là hàm Cauchy) II. M t s tính ch t Tính ch t 1. Hàm f tuy n tính và th a mãn
x ≥ 0 th` f (x) ≥ 0, khi đóf là hàm đ ng bi n. (N u v i m i x ≥ 0 ⇒ f (x) ≤ 0 thì hàm ngh ch bi n).
        i
Ch ng minh Xét x ≤ y ⇒ y − x ≥ 0 ⇒ f (y − x) ≥ 0 Ta có f (y) = f (y − x + x) = f (y − x) + f (x) ≥ f (x).
V y f là hàm tăng. Tính ch t 2. Hàm tuy n tính f là hàm l . Ch ng minh Ta có f (0) = f (0 + 0) =
2f (0) ⇒ f (0) = 0. T đó f (0) = f (x + (−x)) = f (x) + f (−x) = 0 ⇒ f (−x) = −f (x), ∀x ∈ R. V y
f là hàm l . Tính ch t 3. Hàm tuy n tính f liên t c t i x = 0 thì liên t c trên toàn t p s th c R.
Ch ng minh Xét x0 ∈ R b t kỳ, ta có: lim [f (x) − f (x0 )] = lim [f (x) + f (−x0 )] = lim f (x − x0 ) =
                                                         x→x0                     x→x0                   x→x0
lim f (y) = f (0) = f (0) = 0 V y hàm s liên t c t i x0 ∈ R. Do x0 l y b t kỳ trên R nên ch ng t hàm
y→0
s liên t c trên toàn b R. Tính ch t 4. Hàm s f tuy n tính và đ ng bi n trên R thì liên t c trên R.
Ch ng minh Cho y = 0 ⇒ f (x) = f (x) + f (0) ⇒ f (0) = 0 Cho y = x ⇒ f (2x) = 2f (x), b ng quy
n p ta d dàng ch ng minh đư c: f (nx) = nf (x), ∀n ∈ N, ∀x ∈ € Š M t khác t công th c (1) suy
                € Š         € Š
                                                                       R(1)
ra f (x) = nf n hay f n = n f (x), ∀x ∈ R, ∀n ∈ N, do đó: f m x = m f (x), ∀x ∈ R, ∀m, n ∈ N
                 x            x      1
                                                                         n       n
hay f (qx) = qf (x), ∀q ∈ Q, ∀x ∈ R Đ n đây ta có th gi i quy t theo hai cách sau: V i ε  0 b t
                      ε
kỳ, ch n δ = 1+|f (1)|+|f (−1)| , khi đó v i m i x ∈ R, |x|  δ theo tính ch t c a t p s th c thì t n t i
                                                                                           €    Š
m, n ∈ N sao cho |x|  m  δ, t c là − m  x  m . Vì f là hàm đ ng bi n nên f − m  f (x) 
                             n                 n         n                                    n
  € Š
f m ⇒ m f (−1)  f (x)  m f (1) ⇒ − m (1 + |f (1)| + |f (−1)|)  f (x)  m (1 + |f (1)| + |f (−1)|) V y
    n      n                      n          n                                  n
|f (x) − f (0)| = |f (x)|  m (1 + |f (1)| + |f (−1)|)  δ (1 + |f (1)| + |f (−1)|) = ε. Do đó hàm s liên
                                n
t c t i x = 0 nên liên t c trên R Ho c ta có th là như sau: t f (qx) = qf (x), ∀q ∈ Q, ∀x ∈ R nên
f (x) = xf (1), ∀x ∈ Q Hơn n a v i m i x ∈ R, t n t i hai dãy h u t (un ), (vn ) ⊂ Q : un  x  vn
mà lim un = lim vn = x. Do hàm đ ng bi n nên f (un )  f (x)  f (vn ) ⇒ un f (1)  f (x)  vn f (1).
      n→∞            n→∞
Chuy n qua gi i h n ta đư c f (x) = f (1)x ∀x ∈ R hay f (x) = ax nên liên t c trên R. Tính ch t 5.
Hàm tuy n tính f và liên t c trên R có bi u di n là f (x) = ax, (a = f (1)). Ch ng minh Theo cách thi t
l p trong tính ch t 3 ta có f (x) = xf (1), ∀x ∈ Q. Vì v i m i x ∈ R, luôn t n t i dãy {xn }n∈N ⊂ Q sao
cho lim xn = x. Vì f liên t c nên
      n→∞

                                  lim f (xn ) = f (x) ⇒ lim xn f (1) = f (x) ⇒ f (x) = ax
                                  n→∞                           n→∞


,v i
                                                                a = f (1)
, th l i th y hàm s này th a mãn yêu c u bài toán. V y f (x) = ax, ∀x ∈ R. Tính ch t 6. Cho c  0.
N u hàm s f tuy n tính và th a mãn đi u ki n |f (x)| ≤ c ∀x ∈ [−1, 1] thì f (x) = ax v i |a| ≤ c Ch ng
minh T tính ch t 3 ta có f (qx) = qf (x), ∀q ∈ Q, x ∈ R Gi s (xn ) là dãy s th c = 0 th a mãn
 lim xn = 0. V i m i giá tr c a xn ta ch n m t s h u t qn th a mãn: √1 ≤ qn ≤ √1 , n = 1, 2, ...(có
n→∞                                                                                3
                                                                                           |xn |        |xn |
th t giá tr n = n0 , n0 +1, ... đ th a mãn đi u ki n trên) thì ta có: n→∞ qn = ∞ và n→∞ (xn .qn ) = 0 V y
                                                                       lim           lim
1
1
|f (xn )| =
f          .q .x
qn n n
                                  =   qn
                                         |f (qn xn )|,   ∀n ∈ N, do n→∞ (xn .qn ) = 0 nên v i n đ l n thì qn xn ∈ [−1, 1]
                                                                     lim
                                                                    1
nên |f (qn xn )| ≤ c, v i n đ l n. Do đó |f (xn )| ≤               qn
                                                                      c   Do đó lim f (xn ) = 0 = f (0) nên hàm f liên t c
                                                                                n→∞
t i 0, t đó liên t c trên toàn b R do đó có bi u di n f (x) = ax. T đi u ki n bài toán ta đư c hàm c n
tìm là f (x) = ax v i |a| ≤ c Tính ch t 7. N u hàm s f tuy n tính và th a mãn đi u ki n t n t i h ng
s M  0 sao cho f (x) ≤ M ∀x ∈ [0, 1] thìf (x) = ax Ch ng minh T f (qx) = qf (x) ∀q ∈ Q, ∀x ∈ R

GV: Tr n Minh Hi n . . . . . . PTH b i dư ng h c sinh gi i . . . . . . Trư ng THPT chuyên Quang Trung
www.VNMATH.com 2 PHƯƠNG TRÌNH HÀM CAUCHY

hay f (x) = ax ∀x ∈ Q T đi u ki n bài toán ta có: f (1) − f (x) = f (1 − x) ≤ M ∀x ∈ [0, 1], Suy ra
f (1) − M ≤ f (x) ≤ M ∀x ∈ [0, 1] V y t n t i h ng s N  0 mà |f (x)| ≤ N ∀x ∈ [0, 1] ⇒ |f (x)| ≤
N ∀x ∈ [−1, 1](do f (−x) = −f (x)), đ n đây ta có th là ti p theo như tính ch t 6. đây
ta€cóŠth ch ng
minh khác như sau: V i m i x ∈ R, khi đó v i r ∈ Q+ sao cho |x|  r thì
x
≤ 1, do đó
f x
≤ N . Vì
r
r
€    Š
1
x
r
    ∈ Q nên
f       r
= 1 |f (x)| ≤ N ⇒ |f (x)| ≤ r.N Cho r → |x| thì |f (x)| ≤ N |x|. Suy ra lim f (x) = 0
                             r                                                                     x→0
hay f liên t c t i 0 nên liên t c trên toàn b R. Do đó f (x) = ax Nh n xét 1. Cho t p A = R, [0, ∞)
hay (0, ∞). N u f : A → R th a mãn f (x + y) = f (x) + f (y) và f (xy) = f (x)f (y), ∀x, y ∈ A, thì
ho c là f (x) = 0, ∀x ∈ A ho c là f (x) = x, ∀x ∈ A Ch ng minh Theo tính ch t c a hàm c ng tính thì
f (x) = f (1).x, ∀x ∈ Q. N u f (1) = 0 thì f (x) = f (x.1) = f (x).f (1) = 0, ∀x ∈ A. N u f (1) = 0 do
                                                           T                           √     √          √
f (1) = f (1)f (1) ⇒ f (1) = 1 ⇒ f (x) = x, ∀x ∈ A Q N u y ≥ 0 thì f (y) = f ( y)f ( y) = f 2 ( y) ≥ 0
và do đó f (x + y) = f (x) + f (y) ≥ f (x), hay ch ng t f là hàm tăng. Bây gi v i m i x ∈ AQ, theo
tính trù m t c a t p s th c, t n t i hai dãy pn , qn ∈ Q sao cho pn  x  qn ; pn                  x và qn    x,
khi n → ∞. Do f là hàm tăng, ta có: pn = f (pn ) ≤ f (x) ≤ f (qn ) = qn Chuy n qua gi i h n ta có
f (x) = x, ∀x ∈ A III. Các h qu tr c ti p c a hàm Cauchy T quan h cho hàm f liên t c th a mãn
đi u ki n f (x + y) = f (x) + f (y) ta có bi u di n c a hàm là f (x) = ax. N u ta đ t vào quan h hàm trên
qua phép logarit Nepe t c là: ln f (x+y) = ln f (x)+ln f (y) = ln(f (x).f (y)), suy ra f (x+y) = f (x).f (y).
V y n u f (x)  0 v i m i x ∈ R thì quan h hàm f (x + y) = f (x).f (y) d dàng chuy n v quan h hàm
Cauchy qua phép logarit. Tuy nhiên t quan h hàm đó d dàng th y đư c bài toán v n gi i đư c v i
mi n xác đ nh trên R.                                                                         
                                                                                                f (x) ≡ 0
    H qu 1. Các hàm s liên t c trên R th a mãn đi u ki n: f (x+y) = f (x).f (y) (1) là:
                                                                                                f (x) = ax (a  0)
Ch ng minh Nh n th y hàm đ ng nh t f (x) ≡ 0 th a mãn quan h đó. Xét hàm không đ ng nh t 0,
khi đó t n t i x0 :f (x0 ) = 0 thì: f (x0 ) = f ((x0 − x) + x) = Š (x0 − € Š (x) = 0 ⇒ f (x) = 0 ∀x ∈ R Và
                                                                €
                                                                       f    x)f
                                                                  x  x     2 x
cũng th a đi u ki n luôn dương, th t v y: f (x) = f 2 + 2 = f 2  0 ∀x ∈ R Do đó đ n đây ta
ch c n đ t ln f (x) = g(x) thì ta có quan h : g(x + y) = g(x) + g(y) V y g(x) = bx, b ∈ R tùy ý. V y
f (x) = ebx = ax (a  0). V y hai hàm th a mãn quan h đó là:
    B y gi l i t hàm Cauchy nêu ta nâng lũy th a c a bi n lên t x thành ex ta đư c quan h là
f (e ) = f (ex ) + f (ey ) ⇒ g(x + y) = g(x) + g(y) v i g(x) = f (ex ) và hàm g thu đư c l i chính là hàm
    x+y

Cauchy. M t khác t f (ex+y ) = f (ex ) + f (ey ) ⇒ f (ex .ey ) = f (ex ) + f (ey ), bây gi thay ngư c tr l i ex
b i x thì ta đư c quan h m i là f (xy) = f (x) + f (y). Quan h này v i quan h Cauchy tương tác v i
nhau b i vi c nâng lũy th a c a bi n. Tuy nhiên vi c nâng lũy th a c a bi n l i có yêu c u bi n ph i
dương. N u có m t bi n b ng 0 thì bài toán tr nên d dàng v i k t qu là f (x) ≡ 0, n u c hai bi n
cùng dương thì bài toán chuy n v phương trình hàm Cauchy qua phép nâng bi n lên lũy th a. N u c
hai s cùng âm thì tích xy là s dương nên l i quy v trư ng h p hai bi n cùng dương.
    H qu 2. Các hàm s f (x)liên t c trên R{0} th a mãn đi u ki n:f (xy) = f (x) + f (y) ∀x, y ∈ R
(2)là: f (x) = b ln |x| ∀x ∈ R{0}, b ∈ R Ch ng minh N u x = y = 1 thì t (3) ta đư c f (1) = 0. L i
cho x = y = −1 ta đư c f (−1) = 0. Bây gi cho y = −1 thì ta đư c f (x) = f (−x) ∀x ∈ R. Do đó f là
hàm ch n. a) Xét x, y ∈ R+ , đ t x = eu , y = ev , f (eu ) = g(u) ta đư c g(u + v) = g(u) + g(v) ∀u, v ∈ R
⇔ g(t) = bt ⇒ f (x) = a ln x ∀x ∈ R+ , a ∈ R b) N u x, y ∈ R− thì xy ∈ R+ nên v i y = x ta đư c:
f (x) = 2 f (x2 ) = 1 b ln(x2 ) = b ln |x| ∀x ∈ R− , b ∈ R
         1
                     2
    L i ti p t c t quan h hàm f (x + y) = f (x).f (y) ta l i nâng bi n theo lũy th a c a e thì có d ng
f (ex+y ) = f (ex )f (ey ) ⇒ f (ex .ey ) = f (ex )f (ey ) và ta đư c quan h hàm: g(xy) = g(x)g(y) Hi n nhiên
bài toán có ngay l i gi i n u mi n xác đ nh ch a s 0. Do đó ta đ t v n đ đó như sau:
    H qu 3. Các hàm f (x) liên t c trên R{0} th a mãn đi u ki n:f (xy) = f (x)f (y), ∀x, y ∈ R{0}là:



GV: Tr n Minh Hi n . . . . . . PTH b i dư ng h c sinh gi i . . . . . . Trư ng THPT chuyên Quang Trung
www.VNMATH.com 2 PHƯƠNG TRÌNH HÀM CAUCHY

                                                   8

                              α
                                                      xβ , ∀x ∈ R+
f (x) = 0 f (x) = |x| f (x) =                                               Ch ng minh Thay y = 1 ⇒ f (x)(1 − f (1)) =
                                                   :
                                                       − |x|β , ∀x ∈ R−
0, ∀x ∈ R{0} (1)Š N u f (1) €=Š 1 thì t (1) suy ra f (x) ≡ 0, ∀x ∈ R{0} Xét f (1) = 1, khi đó
                €
                    1              1
1 = f (1) = f x. x = f (x)f x , ∀x ∈ R{0}. V y f (x) = 0, x ∈ R{0}. a) Xét x, y ∈ R+ , đ t
x = eu , y = ev va g(t) = f (et ). Khi đó ta có: g(u + v) = g(u)g(v), ∀u, v ∈ R V yg(t) = at ∀t ∈ R(a 
0 tuy y`) và do đó: f (x) = f (eu ) = g(u) = au = aln x = xln a = xα , ∀x ∈ R+ trong đóα = ln a b)
        u
Bây gi ta xét trư ng h  x = 0, y = 0 b t kỳ thì cho và x = y = −t ta nh n đư c f 2 (t) = f (t2 ) =
                          p
                            f (−t) = f (t) = tc (hay 0)
f (−t)f (−t) = f 2 (−t) ⇒                               V y trong trư ng h p t ng quát ta có các nghi m
                            f (−t) = −f (t) = −tc
                                         8
                                                            xβ , ∀x ∈ R+
là: a) f (x) = 0 b) f (x) = |x|α f (x) = :
                                                             − |x|β , ∀x ∈ R−
    T quan h hàm Cauchy f (x + y) = f (x) + f (y) ta th c hi n v trái theo trung bình c ng v trái
theo bi n và trung bình c ng v ph i theo hàm s thì ta nh n đư c: Š                    €
    H qu 4(Hàm Jensen). Các hàm f (x) liên t c trên R th a mãn f x+y = f (x)+f (y) (4) là:f (x) = ax+b
                                  € Š                                            €     Š 2           2
                                    x          f (x)+f (0)        f (x)+f (y)      x+y     f (x+y)+f (0)
Ch ng minh Cho y = 0 ⇒ f 2 =                        2
                                                           . V y:       2
                                                                              =f 2 =             2
                                                                                                         ⇒ f (x + y) + f (0) =
f (x) + f (y) Đ t g(x) = f (x) − f (0) thì ta có g(x + y) = g(x) + g(y) hay g(x) = ax⇒ f (x) = ax + b
    L i trong quan h hàm Jensen ta Š c hi n logarit Nepe n i t i c a bi n(dĩ nhiên trong trư ng
                                       €
                                                    th
                                                                                 √
h p các bi n dương, ta đư c:f            ln x+ln y
                                              2
                                                      = f (ln x)+f (ln y) ⇔ f (ln xy) = f (ln x)+f (ln y) . T v n đ này đ t
                                                                2                                 2
ngư c lŠ i ta đư c h qu sau: H qu 5. Các hàm f (x) xác đ nh và liên t c trên R+ th a mãn đi u ki n:
  €√
f     xy = f (x)+f (y) ∀x, y ∈ R+ (5) là f (x) = a ln x + b Đi u ki n x, y ∈ R+ là đ cho hàm s luôn đư c
                  2
xác đ nh. Ch ng minh Đ t x = eu , y = ev , g(u) = f (eu ). Khi đó g(u) liên t c trên R và th a mãn đi u
        €     Š
ki n: g u+v = g(u)+g(v) ∀u, v ∈ R Suy ra g(u) = au + b ⇒ f (x) = a ln x + b, ∀x ∈ R+ .
           2          2                 €       Š
    Cũng l i t quan h hàm f x+y = f (x)+f (y) n u ta vi t đư c vào dư i d ng c a bi u di n logarit
                                            2
              €     Š                                   € 2 Š            È                 €    Š       È
                x+y     ln f (x)+ln f (y)
t c là: ln f 2        =         2
                                            ⇒ ln f x+y = ln f (x)f (y) ⇒ f x+y = f (x)f (y) T c là ta
                                                            2                                 2
                         €      Š        È
đư c quan h hàm: f x+y = f (x)f (y). V y ta có: H qu 6. Hàm s f : R → R liên t c th a
                              2 2
  €    Š     È                      f (x) ≡ 0
f x+y = f (x)f (y) (6) là:4
     2
                                    f (x) = eax+b (a, b ∈ R)
                                                                                      È
   Ch ng minh T đi u ki n bài toán cho x = y ⇒ f (x) = f 2 (x) ≥ 0. N u t n t i x0 : f (x0 ) = 0 thì:
           È
 €       Š
   x0 +y
f 2 = f (x0 )f (y) = 0 ∀y ∈ R t c là f (x) ≡ 0 N u f (x)  0 thì th c hi n logarit Nepe hai v đưa
v hàm Jensen ta đư c:f (x) = eax+b , a, b tùy ý thu c R. T đó ta có đi u ph i ch ng minh.
   L i t quan h hàm trong h qu 5, th c hi n phép toán ngh ch đ o hàm s (gi s th c hi n
                                    1       1
                    1             f (x)
                                        + f (y)                                 1
đư c) ta có:       √
                f ( xy)
                          =            2
                                                  , b ng cách đ t g(x) =         ta nh n đư c h qu sau: H qu 7. Các
                                                                              f (x)
                                                             +                         √         2
hàm f (x) xác đ nh và liên t c trên R                            th a mãn đi u ki n:f ( xy) = 1 + 1 ∀x, y ∈ R+ (7) là
                                                                                               f (x)   f (y)

hàm h ng f (x) = b ∈ R{0} Ch ng minh T gi thi t bài toán suy ra f (x) = 0 ∀x ∈ R+ . Ta
               1
                   + 1         √
có f (√xy) = f (x) 2 f (y) ⇒ g( xy) = g(x)+g(y) ∀x, y ∈ R+ v i g(x) = f (x) Theo h qu 5 thì g(x) =
       1
                                          2
                                                                        1

a ln x + b ⇒ f (x) = a ln1 . Đ f (x) liên t c trên R+ thì: a ln x + b = 0, ∀x ∈ R+ nên a = 0, b = 0. V y
                         x+b
f (x) = b ∈ R{0}(đpcm).
                                                                                                                           1       1
                                                                                                               1         f (x)
                                                                                                                               + f (y)
    T quan h hàm Jensen n u ta th c hi n ngh ch đ o(v i hàm s ) thì ta có:                       =       f(    x+y   =
                  €       Š
                                                                                    )        2
                                                                                                                2
f (x)+f (y)
 2f (x)f (y)
             hay f x+y = f (x)+f(y) Tuy nhiên đ đ m b o cho phép ngh ch đ o hàm luôn th c hi n đư c
                    2
                         2f (x)f
                                 (y)
thì ta ch c n gi i h n giá tr hàm trong R+ . Do đó ta nh n đư c k t qu : H qu 8. Hàm s f : R → R+

GV: Tr n Minh Hi n . . . . . . PTH b i dư ng h c sinh gi i . . . . . . Trư ng THPT chuyên Quang Trung
www.VNMATH.com 2 PHƯƠNG TRÌNH HÀM CAUCHY

                                €        Š
liên t c th a mãn f x+y = f (x)+f(y) (8) là f (x) = 1 , b  0
                      2
                             2f (x)f
                                     (y)             b
                                         1
    Ch ng minh Ch c n đ t g(x) = f (x) , ta nh n đư c quan h hàm Jensen theo hàm g(x) nêng(x) =
                          1
cx + d. Do đó f (x) = cx+d . Tuy nhiên hàm s này c n ph i th a mãn đi u ki n f (x) ∈ R+ nên:
  1
cx+d
       0, ∀x ∈ R ⇒c = 0, b  0, v y hàm thu đư c là f (x) = 1 , b  0 tùy ý.
                                                              b
    L i v n trong quan h hàm Jensen n u ta th c hi n phép bình phương vào hàm s thì ta nh n ngay
đư c h qu sau:                                            q
                                                 €     Š        2     2
    H qu 9. Hàm s f (x)liên t c trên R th a f x+y = [f (x)] +[f (y)] (9) là f (x) = c v i c ≥ 0. Ch ng
                                                   2              2
                                                                                                        €   €       ŠŠ2                     2               2
minh T quan h hàm s suy ra f (x) ≥ 0, ∀x ∈ R. Ta có: f x+y                             = [f (x)] +[f (y)] . Đ t g(x) = [f (x)]2
                                                                                   2             2          √
thì ta nh n đư c quan h hàm Jensen cho hàm g(x)nên g(x) = ax + b. Do đó f (x) = ax + b. Mà theo
                √
đi u ki n thì ax + b ≥ 0, ∀x ∈ R ⇒ a = 0, b ≥ 0 Ta đư c hàm f (x) = b, b ≥ 0.
     T quan h hàm trong h qu 6, n u ta th È hi n phép nâng lũy th a lên cơ s e(đ i v i bi n) thì
                                                            c
          
            x+y     È                         √
                          x )f (ey ) ⇒ f ( ex .ey ) =
ta có: f e 2 = f (e                                           f (ex )f (ey ) Thay ngư c l i bi n d ng bình thư ng ta
nh n đư c k t qu :                                                                        È
                                                                                     √
     H qu 10. Hàm s f (x) xác đ nh và liên t c trên R+ th a f ( xy) = f (x)f (y), ∀x, y ∈ R+ (10) là:

  f (x) ≡ 0                                                                                                            €   Š

             a                 Ch ng minh Đ t x = eu , y = ev , f (eu ) = g(u) thì ta nh n đư c: g u+v =                 2
  f (x) = c.x , a ∈ R, c  0                                       2
                                     
È                                      g(u) ≡ 0                       f (x) ≡ 0
   g(u)g(v), theo h qu 6 thì:                        au    .Vy4                                                  . Trong quan
                                       g(u) = e + b                   f (x) = ea ln x+b = c.xa , c  0, a ∈ R
                                                                                                           √         2     2
h hàm c a h qu 5, n u ta th c hi n theo quan h hàm bình phương, t c là f 2 ( xy) = f (x)+f (y) ,                         2
th c hi n căn b c hai hai v ta đư c h qu 11. H qu 11. Hàm s f (x) xác đ nh và liên t c trên
                       q
             √              2      2
R+ th a f ( xy) = f (x)+f (y) , ∀x, y ∈ R+ (11) là f (x) ≡ c, c ≥ 0 Ch ng minh T gi thi t c a
                                2
hàm dŠ th y f (x) ≥ 0, ∀x ∈ R+ . Đ t x = eu , y = ev , [f (eu )]2 = g(u). Khi đó g(u) ≥ 0, và ta có:
  €
g u+v = g(u)+g(v) , ∀u, v ∈ R V y g(u) = au + b. Đ g(u) ≥ 0, ∀u ∈ Rthì a = 0, b ≥ 0. Do đó
     2           2
f (x) ≡ c, c ≥ 0.                       €      Š                                                         € Š
     L i t quan h hàm Jensen f x+y = f (x)+f (y) , ta xét phép gán hàm f (x) = g x thì ta nh n đư c
                                            2           2
                                                                                                          1
                                                             
                         1           g ( 1 )+g ( 1 )         2         g ( 1 )+g ( 1 )
quan h hàm s : g (x+y)/2 = x 2 y ⇔ g x+y = x 2 y , thay ngư c tr l i bi n bình thư ng
ta đư c: H qu 12. Hàm s f (x) liên t c trên R{0} th a mãn
                                                      „                   Ž
                                                              2                       f (x) + f (y)
                                                f         1           1           =                 , ∀x, y, x + y = 0
                                                          x
                                                              +       y
                                                                                            2
                      a
(12) là hàm s f (x) = x + b; a, b ∈ R tùy ý. Gi i V i cách thi t l p như trên thì ta có g(x) = ax + b,
            € Š                                                                   €    Š
v i g(x) = f x , khi đó thì f (x) = x + b; a, b ∈ R. L i t quan h hàm Jensen f x+y = f (x)+f (y) , ta
             1                      a
                                                                                    2         2
xét phép gán hàm f (x) = g 11 thì ta nh n đư c quan h hàm:
                           (x)
                                  1            1                  €       Š                                               €       Š              
                                         +                    g       1
                                                                              +g          1         ‚       Œ
                                                                                                                     2g         1
                                                                                                                                        g       1
                  1                1
                                g( x )          1
                                             g( y )                   x                   y           2                         x               y                        2
                       ‹   =                         =               €       Š                 ⇔g            =       €       Š                     =         1            1
          g        1                     2                    2g          1
                                                                                  g       1          x+y            g       1
                                                                                                                                    +g          1
                                                                                                                                                                   1
                                                                                                                                                                g( x )
                                                                                                                                                                         +      1
                                                                                                                                                                             g( y )
                  x+y                                                     x               y                                 x                   y
                   2
                                                                                                                                                                                     ‹
                                                                                                   2
Thay ngư c l i bi n ta đư c: H qu 13. Hàm s f (x) xác đ nh liên t c trên R{0} th a f 1 + 1 =
                                                                                                 x   y
                    2
                             x
                    6
                      f (x) = , a = 0
      2             6        a
          1 (13) là 4                 . B ng cách th c hi n các phép toán khai căn, nâng lũy th a, logarit
  1
f (x)
      + f (y)                1
                      f (x) = , b = 0
                             b
GV: Tr n Minh Hi n . . . . . . PTH b i dư ng h c sinh gi i . . . . . . Trư ng THPT chuyên Quang Trung
www.VNMATH.com 2 PHƯƠNG TRÌNH HÀM CAUCHY

Nepe như trong các ph n trư c ta thu đư c các k t qu tương t sau: H 2    qu 14. Hàm s f (x) xác đ nh
                                 ‹
                              2
                                    È                                     f (x) ≡ 0
liên t c trên R{0} th a f 1 + 1 = f (x)f (y), ∀x, y, x + y = 0(14) là: 4           a               H qu
                            x   y                                         f (x) = e x +b , a, b ∈ R
                                                                            ‹       q
                                                                       2                 [f (x)]2 +[f (y)]2
15. Hàm s f (x) xác đ nh liên t c trên R{0} th a f                   1  1
                                                                        +y
                                                                                 =                2
                                                                                                            , ∀x, y, x
                                                                                                          + y = 0 (15) là:
                                                                                                               √
                                                                      x
                                                                                                                          
                                                                                                   +              x2 +y 2
f (x) ≡ c, c ≥ 0 tùy ý. H qu 16. Các hàm f (x) ≥ 0 xác đ nh liên t c trên R th a f                                 2
                                                                                                                            =
q
          2
   [f (x)] +[f (y)] 2                                  √
                      , ∀x, y ∈ R+ (16) là: f (x) = ax2 + b v i a, b ≥ 0 tùy ý. H qu 17. Các hàm s f (x) xác
            2                             √           
                                               x2 +y 2
đ nh, li n t c trên R và th a f                 2
                                                         = f (x)+f (y) , ∀x, y ∈ R (17) là: f (x) = ax2 + b; ∀a, b ∈ R H
                                                                2
                                                                              √         
                                                                                            È
                                                                                 x2 +y 2
qu 18. Các hàm s f (x) xác đ nh, li n t c trên R th a f                           2
                                                                                           = f (x)f (y), ∀x, y ∈ R (18) là:
2
                                                                                                               √         
  f (x) ≡ 0                                                                                                       x2 +y 2
4                                   H qu 19. Các hàm s f (x) xác đ nh, li n t c trên R th a f                               =
                  2                                                                                                2
  f (x) = eax +b ; ∀a, b ∈ R
      2                                         1
  1
     + 1
              , ∀x, y ∈ R (19) là: f (x) = ax2 +b v i ab ≥ 0, b = 0 tùy ý.
f (x)   f (y)

    IV. Các bài t p v n d ng Bài toán 1. Tìm t t c các hàm f (x) liên t c trên R th a: f (x + y) =
f (x)+f (y)+f (x)f (y) Gi i: T bài toán ta có: f (x+y)+1 = (f (x)+1)(f (y)+1) nên đ t g(x) = f (x)+1
thì ta có g(x+y) = g(x).g(y) ⇒ g(x) = ax v y f (x) = ax −1. Bài toán 2. Tìm t t c các hàm s f (x) liên
t c trên R th a mãn đi u ki n:f (x)+f (y)−f (x+y) = xy, ∀x, y ∈ R Gi i Ta có th vi t l i phương trình
                                                 1
                    f (x) + f (y) − f (x + y) = [(x + y)2 − (x2 + y 2 )]
hàm dư i d ng:                                   2                                            1
                                                                           Đ t g(x) = f (x) + 2 x2 thì ta có
                                1 2            1 2               1       2
                     ⇔ f (x) + x + f (y) + y = f (x + y) + (x + y)
                                2              2                 2
g(x) là hàm liên t c trên R th a mãn đi u ki n: g(x) + g(y) = g(x + y) V y g(x) = ax, ∀x ∈ R, a là m t
                                1
h ng s th c, nên f (x) = − 2 x2 + ax. Th l i th y hàm này th a mãn yêu c u bài toán. Bài toán 3. Cho
a ∈ R, tìm t t c các hàm liên t c f : R → R sao cho: f (x − y) = f (x) − f (y) + axy, ∀x, y ∈ R Gi i Cho
x = 1, y = 0 ⇒ f (1) = f (1) − f (0) nên f (0) = 0. L i cho x = y = 1 ⇒ f (0) = f (1) − f (1) + a ⇒ a = 0.
V y v i a = 0 thì không t n t i hàm s . Ta vi t l i quan h hàm f (x − y) = f (x) − f (y), ∀x, y ∈ R
T đây ta đư c: f (x) = f (x + y − y) = f (x + y) − f (y) ⇒ f (x + y) = f (x) + f (y), x, y ∈ R V y
f (x) = ax, ∀x ∈ R Bài toán 4. Tìm t t c các hàm s f (x) xác đ nh liên t c trên R+ th a mãn đi u
         
ki n:f x = f (x) − f (y) ∀x, y ∈ R+ Gi i Đ t x = t → x = ty thay vào ta có: f (t) = f (ty) − f (y) ⇒
         y                                          y
f (ty) = f (t) + f (y). V y f (x) = a ln x ∀x ∈ R+ , a ∈ R.
    Bài toán 5. Cho a, b ∈ R{0}, tìm các hàm f (x) xác đ nh liên t c trên R và th a mãn đi u ki n:
f (ax + by) = af (x) + bf (y) ∀x, y ∈ R(1) Gi i Cho x = y = 0 vào (1) ta đư c: f (0)(a + b − 1) = 0
N u a + b = 1 thì f (0) = 0. V y đi u ki n Cauchy đư c th a mãn, nên khi đó thì f (ax) = af (x)
và f (bx) = bf (x), và ta có quan h f (ax + by) = f (ax) + f (by), ∀x, y ∈ R. V y f (x) = x. N u
a + b = 1 thì nh n giá tr tùy ý, v y ta ph i đ t m t hàm m i đ đư c quan h Cauchy là g(x) =
f (x) − f (0) thì g(0) = 0 và tương t như ph n trình bày trên ta có f (x) = cx + d V y: f (ax + by) =
                                  
                                    a + b = 1 ⇒ f (x) = cx, c ∈ R
af (x) + bf (y) ∀x, y ∈ R là:                                            Nh n xét: V i cách làm tương t
                                    a + b = 1 ⇒ f (x) = cx + d, c, d ∈ R
cho quan h f (ax + by) = af (x) + bf (y) Bài toán 6. Xác đ nh các hàm s f liên t c trên R th a
mãn đi u ki n:f (2x − y) = 2f (x) − f (y), ∀x, y ∈ R Gi i Đ t g(x) = f (x) − f (0) thì g(0) = 0, t
phương trình trên ta thu đư c: g(2x − y) = 2g(x) − g(y), ∀x, y ∈ R Cho y = 0 ⇒ g(2x) = 2g(x)
và cho x = € ⇒ g(−y)Š = −g(y). Thay vào trên ta đư c: g(2x − y) = g(2x) − g(y), ∀x, y ∈ R V y
               0
                          y
g(x+y) = g 2. x − 1. −1 = g(x)−g(−y) = g(x)+g(y), ∀x, y ∈ R. Do đó: g(x) = ax, x ∈ R, a là s th c
                 2



GV: Tr n Minh Hi n . . . . . . PTH b i dư ng h c sinh gi i . . . . . . Trư ng THPT chuyên Quang Trung
www.VNMATH.com 2 PHƯƠNG TRÌNH HÀM CAUCHY

tùy ý. V y f (x) = ax+b, th l i th y hàm này th a mãn yêu c u bài toán. Bài toán 8(Đ ngh IMO 1979).
Ch ng minh r ng m i hàm f : R → R th a mãn đi u ki n: f (xy+x+y) = f (xy)+f (x)+f (y), ∀x, y ∈ R
khi và ch khi f (x + y) = f (x) + f (y), ∀x, y ∈ R Gi i D th y n u f tuy n tính thì f th a mãn h
th c đ u tiên. Gi s f (xy + x + y) = f (xy) + f (x) + f (y), ∀x, y ∈ R đ t y = u + v + uv ta đư c:
f (x + u + v + xu + xv + uv + xuv) = f (x) + f (u + v + uv) + f (xu + xv + xuv) Hoán đ i vai trò c a x và u
ta đư c: f (u + x + v + ux + uv + xv + uxv) = f (u) + f (x + v + xv) + f (ux + uv + uxv) So sánh hai đ ng
th c trên ta đư c: f (x) + f (u + v + uv) + f (xu + xv + xuv)= f (u) + f (x + v + xv) + f (ux + uv + uxv)
Hay f (uv) + f (xu + xv + xuv) = f (xv) + f (xu + uv + xuv) L y x = 1 ta có f (u) + 2f (uv) =
f (u + 2uv), theo ví d 4 ta có đi u ph i ch ng minh. Bài toán 9. Tìm t t c các hàm s f (x) liên
t c trên R th a mãn đi u ki n:f (x)f (y) − f (x + y) = sin x. sin y, ∀x, y ∈ R Gi i Thay y = 0 ta có
f (x)[f (0) − 1] = 0 ⇒ f (0) = 1, vì d dàng nh n th y f (x) ≡ 0, ∀x ∈ R không là nghi m c a phương
trình. Thay y = −x ta nh n đư c: f (x)f (−x) − f (0) = −sin2 x, €∀x ∈ R ⇒ f (x)f (−x) = 1 − sin2 x =
                                                                   € Š          Š              € Š
cos2 x, ∀x ∈ R(1). Thay x = π vào (1) ta đư c nên: f π .f − π = 0 Ho c f π = 0 thay vào
                                      2                              2        2
                        €      Š                 €       Š                            €       Š 2
hàm ta đư c: −f x + π = sin x ⇒ f x + π = − sin x → f (x) = − sin x − π = cos x, ∀x ∈ R
                             2
          €     Š                                     €2     Š                        €    2Š
Ho c f − π = 0 thay vào hàm ta đư c: f x − π = sin x ⇒ f (x) = sin x + π = cos x, ∀x ∈ R
              2                                            2                                2
D dàng ki m tra l i th y f (x) = cos x là hàm th a mãn yêu c u bài toán. Bài toán 10. Tìm t t c
các hàm s f : R → R th a mãn f (x + y − xy) + f (xy) = f (x) + f (y) (1) v i m i x, y ∈ R. Gi i Ta
ch ng minh n u f là hàm s th a mãn đi u ki n bài toán thì hàm s F (x) = f (x + 1) − f (x) s th a
mãn đi u ki n hàm Cauchy F (u + v) = F (u) + F (v) v i m i (u, v) ∈ ∆ = {(u, v) : u + v  0ho c
u = v = 0 ho c u + v ≤ −4} Th t v y, gi s f là hàm s th a mãn đi u ki n (1). Ta đ nh nghĩa
hàm s f ∗ (x, y) b i: f ∗ (x, y) = f (x) + f (y) − f (xy) D th y r ng hàm f ∗ th a mãn phương trình hàm:
f ∗ (xy, z) + ∗ (x, y) = f (x, yz) + f ∗ (y, z)(1) M t khác ta có f ∗ y) = f (x + y − xy)(2) Thay (2) vào (1)
              f             ∗
                                                           
                                                                       (x,
                      1                                          1                                   1
ta đư c: f xy + y − x + f (x + y − xy) = f (1) + f y + y − 1 , v i m i x, y = 0 Đ t xy + y − x = u + 1
và x + y − xy = v + 1(3) ta nh n đư c: f (u + 1) + f (v + 1) = f (1) + f (u + v + 1), v i m i u, v th a
                                                                               1
mãn đi u ki n trên. B ng vi c c ng hai đ ng th c c a (3) ta có y + y = u + v + 2, đ có nghi m y = 0
ch trong trư ng h p D = {(u + v + 2)2 − 4 = (u + v)(u + v + 4) ≥ 0}. Đi u ki n này x y ra khi và
ch khi ho c là u + v  0 ho c u + v = 0 ho c u + v + 4 ≤ 0. B ng vi c ki m tra đi u ki n ta th y bài
toán đư c th a. N u f là m t nghi m c a bài toán thì f ph i có d ng f (x) = F (x − 1) + f (1)(1) v i
m i x, trong đó F th a mãn phương trình hàm Cauchy F (x + y) = F (x) + F (y) v i m i x, y. Ch ng
minh Theo ch ng minh trên, thì f có d ng v i F th a mãn phương trình Cauchy v i m i (u, v) ∈ ∆.
Ta s ch ng minh r ng F th a mãn phương trình Cauchy v i m i (u, v) b t kỳ. Gi s , khi đó t n t i
m t s th c sao cho các đi m (x, u), (x + u, v), (x, u + v) n m trong ∆ v i vi c xác đ nh x là: c đ nh
(u, v) ∈ ∆ thì t các b t đ ng th c x + u  0, x + u + v  0 ta tìm đư c đi u ki n c a x. Nhưng khi đó:
F (u) = F (x + u) − F (x)
F (v) = F (x + u + v) − F (x + u) Suy ra t các phương trình này ta có F (u) + F (v) = F (u + v). Và bài
F (u + v) = F (x + u + v) − F (x)
toán đư c ch ng minh.
     Bài toán 14(VMO 1992 b ng B). Cho hàm s f : R → R th a mãn f (x + 2xy) = f (x) + 2f (xy),
∀x, y ∈ R. Bi t f (1991) = a, hãy tính f (1992) Gi i Thay x = 0 ta đư c f (0) = 0. Thay y = −1 ta
                                                                        € Š
nh n đư c f (x) = −f (−x). Thay y = − 2 ta đư c f (x) = 2f x . Xét x = 0 và s th c t b t kỳ, đ t
                                                  1
                                                      € Š                2
       t                                               t
y = 2x ta nh n đư c: f (x + t) = f (x) + 2f 2 = f (x) + f (t) V y f là hàm Cauchy nên f (x) = kx,
v i k là h ng s nào đó. T f (1991) = a ⇒ k.1991 = a ⇒ k = 1991 . Do đó f (1992) = 1992 a Bài toán
                                                                            a
                                                                                                   1991
15. Tìm t t c các hàm s f (x) xác đ nh trên (0, +∞), có đ o hàm t i x = 1 và th a mãn đi u ki n
            √            √
f (xy) = xf (y) + yf (x), ∀x, y ∈ R+ Gi i Xét các hàm s sau g(x) = f√x . T gi thi t c a bài toán
                                                                                  (x)
         √                √           √
ta có: xy.g(xy) = xy.g(x) + xy.g(y) ⇔ g(xy) = g(x) + g(y), ∀x, y ∈ R+ V y g(x) = loga x, x  0.

GV: Tr n Minh Hi n . . . . . . PTH b i dư ng h c sinh gi i . . . . . . Trư ng THPT chuyên Quang Trung
www.VNMATH.com 2 PHƯƠNG TRÌNH HÀM CAUCHY

                                       √
T đó ta có k t qu hàm s f (x) = k. x.loga x v i k ∈ R. L i t (1) n u ta đ t z = x + y thì y = z − x
và quan h (1) tr thành f (z) = f (x).f (z − x), n u v i gi thi t f (x) = 0 ∀x ∈ R thì ta có th vi t l i
                     f (z)
như sau: f (z − x) = f (x) , và ta đ xu t đư c bài toán sau đây: Bài toán 18. Xác đ nh các hàm s f (x)
                                      8
                                      
                                       f (x − y) =
                                                    f (x)
                                                          , ∀x, y ∈ R
liên t c trên R th a mãn đi u ki n:                f (y)             (2) Vì gi thi t là f (x) = 0 ∀x ∈ R
                                      :
                                        f (x) = 0 ∀x ∈ R
                              x
nên ch có hàm s f (x) = a (a  0) th a mãn yêu c u bài toán.
    To be continued
    .




GV: Tr n Minh Hi n . . . . . . PTH b i dư ng h c sinh gi i . . . . . . Trư ng THPT chuyên Quang Trung
www.VNMATH.com                   3 PHƯƠNG PHÁP QUY N P


3      Phương pháp quy n p
    Phương pháp này yêu c u ta trư Šc h t tính f (0), f (1) r i d a vào đó tính f (n) v i n ∈ N. Sau đó
                                 €
                                   1
tính f (n) v i n ∈ Z. Tính ti p f n , t đó suy ra bi u th c c a f (r) v i r ∈ Q. Phương pháp này
thư ng s d ng khi c n tìm hàm s xác đ nh trên N, Z, Q.

Ví d 3.1. Tìm t t c các hàm s f : Q → Q th a mãn đi u ki n:

                        f (1) = 2,   f (xy) = f (x)f (y) − f (x + y) + 1, ∀x, y ∈ Q.              (11)

Gi i

Cho y = 1 và s d ng gi thi t f (1) = 2 ta đư c

                                      f (x + 1) = f (x) + 1, ∀x ∈ Q.                              (12)

B ng phương pháp quy n p ta ch ng minh đư c

                                f (x + m) = f (x) + m, ∀x ∈ Q, ∀m ∈ N.                            (13)

Ti p theo ta s l n lư t ch ng minh:

a) f (n) = n + 1, ∀n ∈ N. Th t v y trong (12) cho x = 0 ta tìm đư c f (0) = 1. Gi s ta đã có
   f (k) = k + 1 thì
                             f (k + 1) = f (k) + 1 = k + 1 + 1 = k + 2.

b) Ti p theo ta ch ng minh f (m) = m+1, ∀m ∈ Z. Th t v y, trong (12) cho x = −1 ta đư c f (−1) = 0.
   Trong (11) cho y = −1 thì ta có

                                      f (−x) = −f (x − 1) + 1, ∀x ∈ Q.

    Khi đó v i m ∈ Z, m  0 thì đ t n = −m, khi đó n ∈ N nên s d ng k t qu trên và ph n (a) ta
    đư c
                         f (m) = f (−n) = −f (n − 1) + 1 = −n + 1 = m + 1.
                                                                               
                                                                       1
c) Ti p theo ta ch ng minh f (x) = x + 1, ∀x ∈ Q. Trư c tiên ta tính f   , n ∈ N+ , b ng cách trong
                                                                       n
                      1
   (11) cho x = n, y = ta có
                      n
                                                                 
                                                  1       1
                                     2 = (n + 1)f   −f n+   + 1.
                                                  n       n

    L i theo (13) thì                                        
                                               1    1
                                          f n+   =f   +n
                                               n    n
    thay vào phương trình trên ta đư c
                                               
                                            1   n+1  1
                                          f   =     = + 1.
                                            n    n   n

GV: Tr n Minh Hi n . . . . . . PTH b i dư ng h c sinh gi i . . . . . . Trư ng THPT chuyên Quang Trung
www.VNMATH.com                   3 PHƯƠNG PHÁP QUY N P

                                                                          m
   T đây thì v i x ∈ Q thì x luôn đư c bi u di n dư i d ng x =              , m ∈ Z, n ∈ N+ , do đó
                                                                          n
                                             
                                           m‹
                                 f (x) = f
                                         
                                           n 
                                             1
                                      = f m.
                                             n
                                                                  
                                                 1          1
                                      = f (m).f      −f m+     +1
                                                 n          n
                                                          
                                                   1        1
                                      = (m + 1).     +1 −f     −m+1
                                                   n        n
                                                      
                                                  1      1
                                      = (m + 1)      +1 − −1−m+1
                                                  n      n
                                        m
                                      =    +1=x+1
                                         n
Th l i th y hàm s f (x) = x + 1, ∀x ∈ Q th a mãn yêu c u bài toán.

    Nh n xét: Bài toán trên k t qu không thay đ i n u ta làm trên t p R và không c n cho trư c
f (1). Vi c cho trư c f (1) giúp quá trình quy n p thu n l i hơn. T l i gi i trên ch c n s lý trên t p
s vô t . Tham kh o thêm v bài này trong bài 8.11.
Ví d 3.2. Tìm t t c các hàm s liên t c f : R → R th a mãn
                            f (x + y) + f (x − y) = 2 (f (x) + f (y)) , ∀x, y ∈ R.
Gi i
a) f (0) = 0, th t v y ch c n thay x = y = 0 ta có đư c k t qu .
b) f là hàm ch n. Đ i vai trò gi a x, y trong đi u ki n ta có
                              f (x + y) + f (y − x) = 2 (f (x) + f (y)) , ∀x, y ∈ R.
   Và như v y thì f (x − y) = f (y − x), ∀x, y ∈ R. Do đó f là hàm ch n nên ta ch c n làm vi c trên R+ .
c) f (nx) = n2 f (x), ∀n ∈ N, ∀x ∈ R+ . Th t v y, cho x = y ta đư c
                                             f (2x) = 4f (x), ∀x ∈ R+ .
   Gi s ta đã có f (nx) = n2 f (x), ∀n ∈ N, ∀x ∈ R+ . Khi đó thay y = nx ta đư c
                              f ((n + 1)x) + f (−(n − 1)x) = 2 (f (x) + f (nx)) ,
   hay                                 €                   Š
                      f ((n + 1)x) = 2 f (x) + n2 f (x) − (n − 1)2 f (x) = (n + 1)2 f (x).

d) f (qx) = q 2 f (x), ∀x ∈ R+ , ∀q ∈ Q+ . Th t v y t (c) thì
                                                 ‹
                                   1             x    1
                           f (x) = 2 f (nx) → f     = 2 f (x), ∀n ∈ N, ∀x ∈ R+ .
                                  n              n   n
                        m
   V i q ∈ Q+ thì q =     v i m, n ∈ N, n = 0 nên
                        n
                                        
                                           x‹   2
                                                    ‹
                                                    x   m2
                             f (qx) = f m.    =m f     = 2 f (x) = q 2 f (x).
                                           n        n   n

GV: Tr n Minh Hi n . . . . . . PTH b i dư ng h c sinh gi i . . . . . . Trư ng THPT chuyên Quang Trung
www.VNMATH.com                    3 PHƯƠNG PHÁP QUY N P


e) Do f liên t c trên R+ nên f (x) = ax2 , ∀x ∈ R+ (v i a = f (1)).

Th l i th y hàm s f (x) = ax2 , ∀x ∈ R th a mãn yêu c u bài toán.
    Nh n xét: Quan h bài toán trên chính là đ ng th c hình bình hành quen thu c. Đó là n u →, →
                                                                                           − −
                                                                                           u v
là hai vector thì ta có                                           
                            |→ + →| + |→ − →| = 2 |→| + |→|
                             − − 2
                             u      v     − − 2
                                          u   v         − 2
                                                        u      − 2
                                                                v
B n ch t c a l i gi i là ch ng minh n u hàm f liên t c và th a mãn h ng đ ng th c hình bình hành
thì b t bu c ph i có d ng f (x) = f (1)x2 . Cũng c n lưu ý là đi u ki n liên t c có th thay b ng đi u
ki n đơn đi u c a hàm s .
Ví d 3.3. Tìm t t c các hàm s f : [0, ∞) → R sao cho f đơn đi u và th a mãn đi u ki n
                                              €        Š
                          (f (x) + f (y))2 = f x2 − y 2 + f (2xy), ∀x ≥ y ≥ 0.

Gi i
                                              1
Cho x = y = 0 ta đư c f (0) = 0 ho c f (0) = .
                                              2
                       1                                           1             1
a) Trư ng h p f (0) = , thì thay x = 1, y = 0 ta l i đư c f (1) = − ho c f (1) = .
                       2                                           2             2
                       1                                   1
     (i) N u f (1) = − thì thay x = y = 1 ta đư c f (2) = . Khi đó ta th y f (0)  f (1), f (1)  f (2),
                       2                                   2
         mâu thu n v i tính ch t đơn đi u c a hàm s .
                     1
    (ii) V y f (1) = . Khi đó thay x = y ta đư c
                     2
                                                         €   Š   1
                                           4 (f (x))2 = f 2x2 + .
                                                                 2
        Xét dãy s x1 = 1, xn+1 = 2x2 , thay vào quan h trên ta đư c
                                   n

                                                                 1
                                       4 (f (xn ))2 = f (xn+1 ) + .
                                                                 2
                                       1
        B ng quy n p ta đư c f (xn ) =    v i m i n ∈ Z+ . Vì xn → ∞ và f đơn đi u nên suy ra
                                       2
               1
        f (x) = v i m i x ≥ 0.
               2
b) Trư ng h p f (0) = 0. Khi đó thay y = 0 ta đư c
                                €   Š
                               f x2 = (f (x))2 , ∀x ≥ 0 → f (x) ≥ 0, ∀x ≥ 0.

   Ngoài ra thay x = y ta đư c 4 (f (x))2 = f (2x2 ). K t h p v i đ ng th c trên ta đư c
                                          4f (x) = f (2x), ∀x ≥ 0.
   Trong phương trình hàm ban đ u, đ t x = u + v, y = u − v thì ta đư c
                                                                     €             Š
                             [f (u + v) − f (u − v)]2 = f (4uv) + f 2(u2 − v 2 )
                                                           ”                   —
                                                     = 4 f (2uv) + f (u2 − v 2 )
                                                     = 4 (f (u) + f (v))2 .

GV: Tr n Minh Hi n . . . . . . PTH b i dư ng h c sinh gi i . . . . . . Trư ng THPT chuyên Quang Trung
www.VNMATH.com                                                          3 PHƯƠNG PHÁP QUY N P


   T đây l y căn b c hai ta đư c

                             f (u + v) + f (u − v) = 2 (f (u) + f (v)) , ∀u ≥ v ≥ 0.

   Phương trình hàm này có nghi m là f (x) = f (1)x2 , ∀x ≥ 0. Ngoài ra d dàng tính đư c f (1) = 0
   ho c f (1) = 1.
                                                                               1
K t lu n: Các hàm s th a mãn là f (x) ≡ 0, f (x) ≡                               và f (x) = x2 , ∀x ≥ 0.
                                                                               2
                                                                                                                                            2   2
   Nh n xét: Bài toán trên xu t phát t m t h ng đ ng th c quen thu c là (x2 + y 2 ) = (x2 − y 2 ) +
(2xy)2 . Và đi m m u ch t c a bài toán là tính ch t f (x2 ) = (f (x))2 , đ suy ra f (x) ≥ 0 khi x ≥ 0.

Ví d 3.4. (China 1996) Cho hàm s f : R → R th a mãn đi u ki n:

                       f (x3 + y 3 ) = (x + y)(f 2 (x) − f (x)f (y) + f 2 (y)), ∀x, y ∈ R.

Ch ng minh r ng f (1996x) = 1996f (x), ∀x ∈ R.

Gi i

a) Tính f (0) và thi t l p cho f (x).
   Cho x = y = 0 ta đư c f (0) = 0. Cho y = 0 ta đư c

                                                            f (x3 ) = xf 2 (x).

   Nh n xét: f (x) và x luôn cùng d u. T đây ta có
                                                                               1               1
                                                           f (x) = x 3 f 2 (x 3 ).

b) Thi t l p t p h p t t c các giá tr a mà f (ax) = af (x).
   Đ t S = {a  0 : f (ax) = af (x), ∀x ∈ R}.

       • Rõ ràng 1 ∈ S.
                                           1
       • Ta ch ng t n u a ∈ S thì a 3 ∈ S. Th t v y
                                                                                                                  
                                                                                                           1                1       1
                            axf 2 (x) = af (x3 ) = f (ax3 ) = f (a 3 x)3 = a 3 x.f 2 (a 3 x)
                                   2                           1
                             ⇒ a 3 f 2 (x) = f 2 (a 3 x)
                                   1                       1
                             ⇒ a 3 f (x) = f (a 3 x)

       • N u a, b ∈ S thì a + b ∈ S. Th t v y
                                                                                                          
                                                                   1       1               1           1
                             f ((a + b)x) = f (a 3 x 3 )3 + (b 3 x 3 )3
                                                   h                                                                                    i
                                       1       1               1       1               1           1           1       1        1   1
                              = (a 3 + b 3 ) f 2 (a 3 x 3 ) − f (a 3 x 3 ).f (b 3 x 3 ) + f 2 (b 3 x 3 )
                                                   h                                   i
                                       1       1       2           1       1       2           1               1
                              = (a 3 + b 3 ) a 3 − a 3 b 3 + b 3 x 3 f 2 (x 3 ) = (a + b)f (x).

   B ng quy n p ta ch ng t m i n ∈ N đ u thu c S. Và bài toán ra là trư ng h p đ c bi t v i n = 1996.


GV: Tr n Minh Hi n . . . . . . PTH b i dư ng h c sinh gi i . . . . . . Trư ng THPT chuyên Quang Trung
www.VNMATH.com                       3 PHƯƠNG PHÁP QUY N P


Nh n xét: 1. N u ch đơn thu n ch ng minh k t qu c a bài toán thì có th quy n p tr c ti p. B ng
cách kh o sát như trên ta s th y h t đư c t t c các giá tr c a a  0 mà f (ax) = af (x).

   2. Do yêu c u “đ c bi t” c a bài toán, nên t nhiên ta s nghĩ ngay là có th ch ng minh đi u đó
đúng v i m i s t nhiên, và qua đó, s nghĩ ngay đ n hư ng quy n p.

   3. Vi c suy ra d u c a f (x) cùng d u v i x là quan tr ng, nó giúp ta tri t tiêu bình phương mà
không c n xét d u, đây cũng là m t đi u đáng lưu ý trong r t nhi u bài t p khác.

   4. Bài toán trên r t có th xu t phát t h ng đ ng th c x3 + y 3 = (x + y) (x2 − xy + y 2 ).

Ví d 3.5. Tìm t t c các hàm f : Z → Z th a mãn:

                         f (x3 + y 3 + z 3 ) = f 3 (x) + f 3 (y) + f 3 (z), ∀x, y, z ∈ Z

    Hint:
1. Tính f (0) và ch ng minh f là hàm l .
2. Ch ng t f (2) = 2f (1), f (3) = 3f (1). Ch ng minh b ng quy n p f (n) = nf (1), ∀n ∈ Z 3. Trong
ch ng minh chuy n t n = k ≥ 0 sang n = k + 1, ta s d ng h ng đ ng th c sau: N u k ch n thì k = 2t,
ta có:
                     (2t + 1)3 + 53 + 13 = (2t − 1)3 + (t + 4)3 + (4 − t)3 khi k = 2t
và n u k l thì k = 2t − 1 khi đó n = 2t luôn đư c vi t dư i d ng 2t = 2j (2i + 1), và đ ng th c trên ch
c n nhân cho 23j

Ví d 3.6. Tìm t t c các hàm f : N → N th a mãn các đi u ki n:

                         f (1)  0 và f (m2 + n2 ) = f 2 (m) + f 2 (n), ∀m, n ∈ N

    Hint:
1. Tính f (0) ⇒ f (m2 + n2 ) = f (m2 ) + f (n2 )
2. Ch ng minh f (n) = n, ∀n ≤ 10. V i n  10 ta s d ng các đ ng th c sau:

                                 (5k + 1)2 + 22 = (4k + 2)2 + (3k − 1)2

                                 (5k + 2)2 + 12 = (4k + 1)2 + (3k + 2)2
                                 (5k + 3)2 + 12 = (4k + 3)2 + (3k + 1)2
                                 (5k + 4)2 + 22 = (4k + 2)2 + (3k + 4)2
                                    (5k + 5)2 = (4k + 4)2 + (3k + 3)2




GV: Tr n Minh Hi n . . . . . . PTH b i dư ng h c sinh gi i . . . . . . Trư ng THPT chuyên Quang Trung
www.VNMATH.com
       4 KHAI THÁC TÍNH CH T ĐƠN ÁNH, TOÀN ÁNH, SONG ÁNH, CH N L C A HÀM S


4      Khai thác tính ch t đơn ánh, toàn ánh, song ánh, ch n l
       c a hàm s
    Trư c tiên ta nh c l i các khái ni m cơ b n này.
a) N u f : R → R là đơn ánh thì t f (x) = f (y) ta suy ra đư c x = y.

b) N u f : R → R là toàn ánh thì v i m i y ∈ R, t n t i x ∈ R đ f (x) = y.

c) N u f : R → R là song ánh thì ta có c hai đ c trưng trên.
N u m t hàm s mà đơn ánh chúng ta r t hay dùng th thu t tác đ ng f vào c hai v , n u m t hàm
f toàn ánh ta hay dùng: T n t i m t s b sao cho f (b) = 0, sau đó tìm b. N u quan h hàm là hàm b c
nh t c a bi n v ph i thì có th nghĩ t i hai quan h này.
Ví d 4.1. Tìm t t c các hàm s f : Q → Q th a mãn

                                    f (f (x) + y) = x + f (y), ∀x, y ∈ Q.

Gi i

Nh n xét, hàm đ ng nh t 0 không th a mãn bài toán. Xét f (x) ≡ 0.
a) f đơn ánh, th t v y, n u f (x1 ) = f (x2 ) thì

                    f (f (x2 ) + y) = f (f (x2 ) + y) → x1 + f (y) = x2 + f (y) → x1 = x2 .

b) f toàn ánh, th t v y, vì t n t i y0 sao cho f (y0 ) = 0. Do đó v ph i c a đi u ki n là m t hàm s b c
   nh t c a x nên có t p giá tr là Q.

c) Tính f (0), cho x = y = 0 và s d ng tính đơn ánh ta đư c

                                         f (f (0)) = f (0) → f (0) = 0.

    T đó thay y = 0 ta đư c
                                             f (f (x)) = x, ∀x ∈ Q.

d) Thay x b i f (x) và s d ng k t qu trên(và đi u này đúng cho v i m i x ∈ Q vì f là toán ánh) thì

                                      f (x + y) = f (x) + f (y), ∀x, y ∈ Q.

T đây ta đư c f (x) = ax thay vào bài toán ta nh n f (x) ≡ x ho c f (x) ≡ −x trên Q.

   Nh n xét: N u yêu c u bài toán trên t p R thì c n thêm tính ch t đơn đi u ho c liên t c. C th ,
các b n có th gi i l i bài toán sau (THTT, 2010): Tìm t t c các hàm s liên t c f : R → R th a
mãn đi u ki n
                                 f (x + f (y)) = 2y + f (x), ∀x, y ∈ R.
Ví d 4.2. Tìm t t c các hàm s f : R → R th a mãn

                                  f (xf (y) + x) = xy + f (x), ∀x, y ∈ R.

GV: Tr n Minh Hi n . . . . . . PTH b i dư ng h c sinh gi i . . . . . . Trư ng THPT chuyên Quang Trung
www.VNMATH.com
       4 KHAI THÁC TÍNH CH T ĐƠN ÁNH, TOÀN ÁNH, SONG ÁNH, CH N L C A HÀM S


Gi i

Thay x = 1 vào đi u ki n hàm ta đư c

                                   f (f (y) + 1) = y + f (1), ∀y ∈ R.

T đây suy ra f là m t song ánh. L y x = 1, y = 0 ta đư c

                            f (f (0) + 1) = f (1) → f (0) = 0 do f đơn ánh.

                              f (x)
Bây gi v i x = 0, đ t y = −         thay vào đi u ki n hàm ta đư c
                                x
                         f (xf (y) + x = 0 = f (0)) → xf (y) = x do f đơn ánh,

hay f (y) = −1, t c là               ‚         Œ
                                       f (x)
                                   f −             = f (y) = −1 = f (b),
                                         x
v i b là m t s th c nào đó(do f là m t toàn ánh). V y f (x) = −bx, ∀x = 0. K t h p v i f (0) = 0
thì vi t g p thành f (x) = −bx, ∀x ∈ R. Thay vào đi u ki n hàm s ta có đư c hai hàm th a mãn là
f (x) ≡ x và f (x) ≡ −x.

   Nh n xét: Bài toán này có th gi i b ng cách th bi n như sau mà không c n dùng đ n tính song
ánh c a hàm s . Thay x = 1 ta đư c

                                   f (f (y) + 1) = y + f (1), ∀y ∈ R.

Ví d 4.3. (Đ ngh IMO 1988) Xác đ nh hàm s f : N → N th a mãn đi u ki n sau:

                                 f (f (n) + f (m)) = m + n, ∀m, n ∈ N.                           (14)

Gi i

a) Trư c tiên ta ki m tra f đơn ánh. Th t v y gi s f (n) = f (m), khi đó

                                    f (2f (n)) = f (f (n) + f (n)) = 2n,

   và
                                   f (2f (n)) = f (f (m) + f (m)) = 2m.
   Do đó m = n, nên f đơn ánh.

b) Ta tính f (f (n)) theo các bư c sau: cho m = n = 0 trong (14) thì ta đư c f (2f (0)) = 0, l i cho
   m = 2f (0) vào trong (14) thì ta đư c

                                          f (f (n)) = n + 2f (0).




GV: Tr n Minh Hi n . . . . . . PTH b i dư ng h c sinh gi i . . . . . . Trư ng THPT chuyên Quang Trung
www.VNMATH.com
       4 KHAI THÁC TÍNH CH T ĐƠN ÁNH, TOÀN ÁNH, SONG ÁNH, CH N L C A HÀM S


c) Tác đ ng f vào c hai v c a (14) và s d ng k t qu trên, ta đư c

                                f (f (f (n) + f (m))) = f (n) + f (m) + 2f (0).

   Ngoài ra theo quan h đ bài thì

                                      f (f (f (n) + f (m))) = f (n + m).

   T đây ta có
                                     f (n + m) = f (n) + f (m) + 2f (0).
   Cho m = n = 0 thì f (0) = 0, do đó quan h trên tr thành hàm c ng tính. V y f (n) = an. Thay
   vào quan h bài toán ta đư c
                                         f (n) = n, ∀n ∈ N.

- Nh n xét: Quan h đơn ánh c a bài toán này không c n thi t trong l i gi i. Và bài toán này có th
ch ng minh b ng quy n p trên N.

    Cách 2. N u xét trên Z+ thì ta có th ch ng minh b ng quy n p f (x) = x, ∀x ∈ N. T c là, dùng
phương pháp, ta ch ng minh không còn t n t i hàm s nào khác. Trư c tiên ta tính f (1). Gi s
f (1) = t  1, đ t s = f (t − 1)  0. Nh n th y r ng n u f (m) = n thì

                                    f (2n) = f (f (m) + f (m)) = 2m.

Như v y
                                        f (2t) = 2, f (2s) = 2t − 2.
Nhưng khi đó thì
                            2s + 2t = f (f (2s) + f (2t)) = f (2t) = 2 → t  1,
đi u này vô lý. V y f (1) = 1. Gi s ta có f (n) = n thì

                                  f (n + 1) = f (f (n) + f (1)) = n + 1.

V y f (n) = n, ∀n ∈ Z+ .
Ví d 4.4. (Balkan 2000) Tìm t t c các hàm s f : R → R th a mãn đi u ki n:

                               f (xf (x) + f (y)) = (f (x))2 + y, ∀x, y ∈ R.                          (15)

Gi i

a) Ta tính f (f (y)) b ng cách cho x = 0 vào (15) ta đư c

                                       f (f (y)) = (f (0))2 + y, ∀y ∈ R.

b) Ch ng t f đơn ánh. Th t v y n u f (y1 ) = f (y2 ) thì f (f (y1 )) = f (f (y2 )). T đây theo ph n (a) thì

                                    f 2 (0) + y1 = (f (0))2 + y2 ⇒ y1 = y2 .

c) Ch ng t f toàn ánh vì v ph i c a (15) là m t hàm b c nh t c a y nên có t p giá tr b ng R. K t
   h p hai đi u trên ta thu đư c f là m t song ánh t R vào R.

GV: Tr n Minh Hi n . . . . . . PTH b i dư ng h c sinh gi i . . . . . . Trư ng THPT chuyên Quang Trung
[Vnmath.com] 13-ki-thuat-giai-phuong-trinh-ham
[Vnmath.com] 13-ki-thuat-giai-phuong-trinh-ham
[Vnmath.com] 13-ki-thuat-giai-phuong-trinh-ham
[Vnmath.com] 13-ki-thuat-giai-phuong-trinh-ham
[Vnmath.com] 13-ki-thuat-giai-phuong-trinh-ham
[Vnmath.com] 13-ki-thuat-giai-phuong-trinh-ham
[Vnmath.com] 13-ki-thuat-giai-phuong-trinh-ham
[Vnmath.com] 13-ki-thuat-giai-phuong-trinh-ham
[Vnmath.com] 13-ki-thuat-giai-phuong-trinh-ham
[Vnmath.com] 13-ki-thuat-giai-phuong-trinh-ham
[Vnmath.com] 13-ki-thuat-giai-phuong-trinh-ham
[Vnmath.com] 13-ki-thuat-giai-phuong-trinh-ham
[Vnmath.com] 13-ki-thuat-giai-phuong-trinh-ham
[Vnmath.com] 13-ki-thuat-giai-phuong-trinh-ham
[Vnmath.com] 13-ki-thuat-giai-phuong-trinh-ham
[Vnmath.com] 13-ki-thuat-giai-phuong-trinh-ham
[Vnmath.com] 13-ki-thuat-giai-phuong-trinh-ham
[Vnmath.com] 13-ki-thuat-giai-phuong-trinh-ham
[Vnmath.com] 13-ki-thuat-giai-phuong-trinh-ham
[Vnmath.com] 13-ki-thuat-giai-phuong-trinh-ham

More Related Content

What's hot

Bdt đánh giá trên biên nhìn vào điểm nút
Bdt đánh giá trên biên nhìn vào điểm nútBdt đánh giá trên biên nhìn vào điểm nút
Bdt đánh giá trên biên nhìn vào điểm nútThế Giới Tinh Hoa
 
Bài tập về cấp của một số nguyên modulo n
Bài tập về cấp của một số nguyên modulo nBài tập về cấp của một số nguyên modulo n
Bài tập về cấp của một số nguyên modulo nLuu Tuong
 
Anh Xa Lien Tuc Tren Khong Gian Topo
Anh Xa Lien Tuc Tren Khong Gian TopoAnh Xa Lien Tuc Tren Khong Gian Topo
Anh Xa Lien Tuc Tren Khong Gian Topoipaper
 
Một số vấn đề về không gian Sobolev
Một số vấn đề về không gian SobolevMột số vấn đề về không gian Sobolev
Một số vấn đề về không gian Sobolevnataliej4
 
Diophantine equations Phương trình diophant
Diophantine equations Phương trình diophantDiophantine equations Phương trình diophant
Diophantine equations Phương trình diophantBui Loi
 
Chuyên đề phương tích và ứng dụng
Chuyên đề phương tích và ứng dụngChuyên đề phương tích và ứng dụng
Chuyên đề phương tích và ứng dụnglovemathforever
 
Cđ giải hpt không mẫu mực
Cđ giải hpt không mẫu mựcCđ giải hpt không mẫu mực
Cđ giải hpt không mẫu mựcCảnh
 
BĐT Côsi ngược dấu
BĐT Côsi ngược dấuBĐT Côsi ngược dấu
BĐT Côsi ngược dấunhankhangvt
 
Ứng dụng đồng dư vào giải toán chia hết lớp 9
Ứng dụng đồng dư vào giải toán chia hết lớp 9Ứng dụng đồng dư vào giải toán chia hết lớp 9
Ứng dụng đồng dư vào giải toán chia hết lớp 9youngunoistalented1995
 
Các chuyên đề bồi dưỡng HSG môn Toán THCS hay nhất
Các chuyên đề bồi dưỡng HSG môn Toán THCS hay nhấtCác chuyên đề bồi dưỡng HSG môn Toán THCS hay nhất
Các chuyên đề bồi dưỡng HSG môn Toán THCS hay nhấtBồi dưỡng Toán lớp 6
 
Bộ sưu tập bất đẳng thức của võ quốc bá cẩn
Bộ sưu tập bất đẳng thức của võ quốc bá cẩnBộ sưu tập bất đẳng thức của võ quốc bá cẩn
Bộ sưu tập bất đẳng thức của võ quốc bá cẩnThế Giới Tinh Hoa
 
Giải một số phương trình nghiệm nguyên trong đề thi toán 9
Giải một số phương trình nghiệm nguyên trong đề thi toán 9Giải một số phương trình nghiệm nguyên trong đề thi toán 9
Giải một số phương trình nghiệm nguyên trong đề thi toán 9Nhập Vân Long
 
Cân bằng hệ số trong bđt AM-GM
Cân bằng hệ số trong bđt AM-GMCân bằng hệ số trong bđt AM-GM
Cân bằng hệ số trong bđt AM-GMNguyễn Việt Long
 

What's hot (20)

Luận văn: Một số lớp bài toán về phương trình hàm, HAY, 9đ
Luận văn: Một số lớp bài toán về phương trình hàm, HAY, 9đLuận văn: Một số lớp bài toán về phương trình hàm, HAY, 9đ
Luận văn: Một số lớp bài toán về phương trình hàm, HAY, 9đ
 
Scp mod p
Scp mod pScp mod p
Scp mod p
 
Bdt đánh giá trên biên nhìn vào điểm nút
Bdt đánh giá trên biên nhìn vào điểm nútBdt đánh giá trên biên nhìn vào điểm nút
Bdt đánh giá trên biên nhìn vào điểm nút
 
Bdt thuần nhất
Bdt thuần nhấtBdt thuần nhất
Bdt thuần nhất
 
Bài tập về cấp của một số nguyên modulo n
Bài tập về cấp của một số nguyên modulo nBài tập về cấp của một số nguyên modulo n
Bài tập về cấp của một số nguyên modulo n
 
Dãy số vmo2009
Dãy số vmo2009Dãy số vmo2009
Dãy số vmo2009
 
9 drichle
9 drichle9 drichle
9 drichle
 
Anh Xa Lien Tuc Tren Khong Gian Topo
Anh Xa Lien Tuc Tren Khong Gian TopoAnh Xa Lien Tuc Tren Khong Gian Topo
Anh Xa Lien Tuc Tren Khong Gian Topo
 
Một số vấn đề về không gian Sobolev
Một số vấn đề về không gian SobolevMột số vấn đề về không gian Sobolev
Một số vấn đề về không gian Sobolev
 
Diophantine equations Phương trình diophant
Diophantine equations Phương trình diophantDiophantine equations Phương trình diophant
Diophantine equations Phương trình diophant
 
Phương trình hàm đa thức
Phương trình hàm đa thứcPhương trình hàm đa thức
Phương trình hàm đa thức
 
Chuyên đề phương tích và ứng dụng
Chuyên đề phương tích và ứng dụngChuyên đề phương tích và ứng dụng
Chuyên đề phương tích và ứng dụng
 
Cđ giải hpt không mẫu mực
Cđ giải hpt không mẫu mựcCđ giải hpt không mẫu mực
Cđ giải hpt không mẫu mực
 
BĐT Côsi ngược dấu
BĐT Côsi ngược dấuBĐT Côsi ngược dấu
BĐT Côsi ngược dấu
 
Ứng dụng đồng dư vào giải toán chia hết lớp 9
Ứng dụng đồng dư vào giải toán chia hết lớp 9Ứng dụng đồng dư vào giải toán chia hết lớp 9
Ứng dụng đồng dư vào giải toán chia hết lớp 9
 
Các chuyên đề bồi dưỡng HSG môn Toán THCS hay nhất
Các chuyên đề bồi dưỡng HSG môn Toán THCS hay nhấtCác chuyên đề bồi dưỡng HSG môn Toán THCS hay nhất
Các chuyên đề bồi dưỡng HSG môn Toán THCS hay nhất
 
Bộ sưu tập bất đẳng thức của võ quốc bá cẩn
Bộ sưu tập bất đẳng thức của võ quốc bá cẩnBộ sưu tập bất đẳng thức của võ quốc bá cẩn
Bộ sưu tập bất đẳng thức của võ quốc bá cẩn
 
Giải một số phương trình nghiệm nguyên trong đề thi toán 9
Giải một số phương trình nghiệm nguyên trong đề thi toán 9Giải một số phương trình nghiệm nguyên trong đề thi toán 9
Giải một số phương trình nghiệm nguyên trong đề thi toán 9
 
Luận văn: Lý thuyết đồ thị với các bài toán phổ thông, HAY, 9đ
Luận văn: Lý thuyết đồ thị với các bài toán phổ thông, HAY, 9đLuận văn: Lý thuyết đồ thị với các bài toán phổ thông, HAY, 9đ
Luận văn: Lý thuyết đồ thị với các bài toán phổ thông, HAY, 9đ
 
Cân bằng hệ số trong bđt AM-GM
Cân bằng hệ số trong bđt AM-GMCân bằng hệ số trong bđt AM-GM
Cân bằng hệ số trong bđt AM-GM
 

Viewers also liked

Bộ sách về phương trình hàm
Bộ sách về phương trình hàmBộ sách về phương trình hàm
Bộ sách về phương trình hàmThế Giới Tinh Hoa
 
Chuyên đề phương trình vô tỷ
Chuyên đề phương trình vô tỷChuyên đề phương trình vô tỷ
Chuyên đề phương trình vô tỷtuituhoc
 
Hướng dẫn giải bài tập chuỗi - Toán cao cấp
Hướng dẫn giải bài tập chuỗi - Toán cao cấpHướng dẫn giải bài tập chuỗi - Toán cao cấp
Hướng dẫn giải bài tập chuỗi - Toán cao cấpVan-Duyet Le
 
Bài tập sử dụng công thức nguyên hàm, tích phân
Bài tập sử dụng công thức nguyên hàm, tích phânBài tập sử dụng công thức nguyên hàm, tích phân
Bài tập sử dụng công thức nguyên hàm, tích phânThế Giới Tinh Hoa
 

Viewers also liked (10)

Bộ sách về phương trình hàm
Bộ sách về phương trình hàmBộ sách về phương trình hàm
Bộ sách về phương trình hàm
 
Dongluan
DongluanDongluan
Dongluan
 
Tcct3 chuoi
Tcct3 chuoiTcct3 chuoi
Tcct3 chuoi
 
Tpds
TpdsTpds
Tpds
 
Dongluanct
DongluanctDongluanct
Dongluanct
 
Bai 2
Bai 2Bai 2
Bai 2
 
Chuyên đề phương trình vô tỷ
Chuyên đề phương trình vô tỷChuyên đề phương trình vô tỷ
Chuyên đề phương trình vô tỷ
 
Chuyên đề về số phức
Chuyên đề về số phứcChuyên đề về số phức
Chuyên đề về số phức
 
Hướng dẫn giải bài tập chuỗi - Toán cao cấp
Hướng dẫn giải bài tập chuỗi - Toán cao cấpHướng dẫn giải bài tập chuỗi - Toán cao cấp
Hướng dẫn giải bài tập chuỗi - Toán cao cấp
 
Bài tập sử dụng công thức nguyên hàm, tích phân
Bài tập sử dụng công thức nguyên hàm, tích phânBài tập sử dụng công thức nguyên hàm, tích phân
Bài tập sử dụng công thức nguyên hàm, tích phân
 

Similar to [Vnmath.com] 13-ki-thuat-giai-phuong-trinh-ham

Phương pháp số và lập trình - Nội suy, Đạo hàm, Tích phân
Phương pháp số và lập trình - Nội suy, Đạo hàm, Tích phânPhương pháp số và lập trình - Nội suy, Đạo hàm, Tích phân
Phương pháp số và lập trình - Nội suy, Đạo hàm, Tích phânHajunior9x
 
ôN thi tốt nghiep thpt-montoan-theo dan gbai - truonghocso.com
ôN thi tốt nghiep thpt-montoan-theo dan gbai - truonghocso.comôN thi tốt nghiep thpt-montoan-theo dan gbai - truonghocso.com
ôN thi tốt nghiep thpt-montoan-theo dan gbai - truonghocso.comThế Giới Tinh Hoa
 
Notes for Optimization Chapter 1 and 2
Notes for Optimization Chapter 1 and 2Notes for Optimization Chapter 1 and 2
Notes for Optimization Chapter 1 and 2Vu Pham
 
Chuyên đề luyện thi đạo học
Chuyên đề luyện thi đạo họcChuyên đề luyện thi đạo học
Chuyên đề luyện thi đạo họcTít Thiện
 
Chuyên đề luyện thi đại học môn toán năm 2014
Chuyên đề luyện thi đại học môn toán năm 2014Chuyên đề luyện thi đại học môn toán năm 2014
Chuyên đề luyện thi đại học môn toán năm 2014Huynh ICT
 
Chuyên đề luyện thi Đại học 2014
Chuyên đề luyện thi Đại học 2014Chuyên đề luyện thi Đại học 2014
Chuyên đề luyện thi Đại học 2014tuituhoc
 
Chuyen de on_thi_cao_hoc_2012_ham_so_va_cuc_tri_1998
Chuyen de on_thi_cao_hoc_2012_ham_so_va_cuc_tri_1998Chuyen de on_thi_cao_hoc_2012_ham_so_va_cuc_tri_1998
Chuyen de on_thi_cao_hoc_2012_ham_so_va_cuc_tri_1998Zome VN
 
Thi thử toán mai anh tuấn th 2012 lần 1 k b
Thi thử toán mai anh tuấn th 2012 lần 1 k bThi thử toán mai anh tuấn th 2012 lần 1 k b
Thi thử toán mai anh tuấn th 2012 lần 1 k bThế Giới Tinh Hoa
 
Phongmath pp khu dang vo dinh
Phongmath   pp khu dang vo dinhPhongmath   pp khu dang vo dinh
Phongmath pp khu dang vo dinhphongmathbmt
 
Pvt dao ham da 11
Pvt dao ham da 11Pvt dao ham da 11
Pvt dao ham da 1114149201
 
Tổng hợp công thức giải nhanh trắc nghiệm toán THPT Quốc gia 2018
Tổng hợp công thức giải nhanh trắc nghiệm toán THPT Quốc gia 2018Tổng hợp công thức giải nhanh trắc nghiệm toán THPT Quốc gia 2018
Tổng hợp công thức giải nhanh trắc nghiệm toán THPT Quốc gia 2018Maloda
 

Similar to [Vnmath.com] 13-ki-thuat-giai-phuong-trinh-ham (20)

Quan2017
Quan2017Quan2017
Quan2017
 
Phương pháp số và lập trình - Nội suy, Đạo hàm, Tích phân
Phương pháp số và lập trình - Nội suy, Đạo hàm, Tích phânPhương pháp số và lập trình - Nội suy, Đạo hàm, Tích phân
Phương pháp số và lập trình - Nội suy, Đạo hàm, Tích phân
 
Luận văn: Phép biến đổi phân tuyến tính, HAY, 9đ
Luận văn: Phép biến đổi phân tuyến tính, HAY, 9đLuận văn: Phép biến đổi phân tuyến tính, HAY, 9đ
Luận văn: Phép biến đổi phân tuyến tính, HAY, 9đ
 
ôN thi tốt nghiep thpt-montoan-theo dan gbai - truonghocso.com
ôN thi tốt nghiep thpt-montoan-theo dan gbai - truonghocso.comôN thi tốt nghiep thpt-montoan-theo dan gbai - truonghocso.com
ôN thi tốt nghiep thpt-montoan-theo dan gbai - truonghocso.com
 
Notes for Optimization Chapter 1 and 2
Notes for Optimization Chapter 1 and 2Notes for Optimization Chapter 1 and 2
Notes for Optimization Chapter 1 and 2
 
Chuyên đề luyện thi đạo học
Chuyên đề luyện thi đạo họcChuyên đề luyện thi đạo học
Chuyên đề luyện thi đạo học
 
Luận văn: Phương pháp giải bài toán cực trị và ứng dụng, HAY
Luận văn: Phương pháp giải bài toán cực trị và ứng dụng, HAYLuận văn: Phương pháp giải bài toán cực trị và ứng dụng, HAY
Luận văn: Phương pháp giải bài toán cực trị và ứng dụng, HAY
 
Luận văn: Phương pháp giải bài toán cực trị, HAY
Luận văn: Phương pháp giải bài toán cực trị, HAYLuận văn: Phương pháp giải bài toán cực trị, HAY
Luận văn: Phương pháp giải bài toán cực trị, HAY
 
Chuyên đề luyện thi đại học môn toán năm 2014
Chuyên đề luyện thi đại học môn toán năm 2014Chuyên đề luyện thi đại học môn toán năm 2014
Chuyên đề luyện thi đại học môn toán năm 2014
 
CHUYÊN ĐỀ: LTĐH TOÁN 2014
CHUYÊN ĐỀ: LTĐH TOÁN 2014CHUYÊN ĐỀ: LTĐH TOÁN 2014
CHUYÊN ĐỀ: LTĐH TOÁN 2014
 
Chuyên đề luyện thi Đại học 2014
Chuyên đề luyện thi Đại học 2014Chuyên đề luyện thi Đại học 2014
Chuyên đề luyện thi Đại học 2014
 
Chuyen de on_thi_cao_hoc_2012_ham_so_va_cuc_tri_1998
Chuyen de on_thi_cao_hoc_2012_ham_so_va_cuc_tri_1998Chuyen de on_thi_cao_hoc_2012_ham_so_va_cuc_tri_1998
Chuyen de on_thi_cao_hoc_2012_ham_so_va_cuc_tri_1998
 
Thi thử toán mai anh tuấn th 2012 lần 1 k b
Thi thử toán mai anh tuấn th 2012 lần 1 k bThi thử toán mai anh tuấn th 2012 lần 1 k b
Thi thử toán mai anh tuấn th 2012 lần 1 k b
 
Ôn thi Toán
Ôn thi ToánÔn thi Toán
Ôn thi Toán
 
Phongmath pp khu dang vo dinh
Phongmath   pp khu dang vo dinhPhongmath   pp khu dang vo dinh
Phongmath pp khu dang vo dinh
 
Pvt dao ham da 11
Pvt dao ham da 11Pvt dao ham da 11
Pvt dao ham da 11
 
Tổng hợp công thức giải nhanh trắc nghiệm toán THPT Quốc gia 2018
Tổng hợp công thức giải nhanh trắc nghiệm toán THPT Quốc gia 2018Tổng hợp công thức giải nhanh trắc nghiệm toán THPT Quốc gia 2018
Tổng hợp công thức giải nhanh trắc nghiệm toán THPT Quốc gia 2018
 
Chuyen de dao ham
Chuyen de dao ham Chuyen de dao ham
Chuyen de dao ham
 
Luận văn: Bất đẳng thức trong lớp hàm siêu việt, HAY, 9đ
Luận văn: Bất đẳng thức trong lớp hàm siêu việt, HAY, 9đLuận văn: Bất đẳng thức trong lớp hàm siêu việt, HAY, 9đ
Luận văn: Bất đẳng thức trong lớp hàm siêu việt, HAY, 9đ
 
Tichchap
TichchapTichchap
Tichchap
 

More from Duy Duy

Bai tap a2 c2
Bai tap a2   c2Bai tap a2   c2
Bai tap a2 c2Duy Duy
 
Ngan hang a2 c2 ths. cao xuan phuong
Ngan hang a2 c2 ths. cao xuan phuongNgan hang a2 c2 ths. cao xuan phuong
Ngan hang a2 c2 ths. cao xuan phuongDuy Duy
 
Bt toan a2
Bt toan   a2Bt toan   a2
Bt toan a2Duy Duy
 
Bai tap a2 c2
Bai tap a2   c2Bai tap a2   c2
Bai tap a2 c2Duy Duy
 
Bt toan cao cap tap 1 nguyen thuy thanh
Bt toan cao cap tap 1 nguyen thuy thanhBt toan cao cap tap 1 nguyen thuy thanh
Bt toan cao cap tap 1 nguyen thuy thanhDuy Duy
 
Da hoa b
Da hoa bDa hoa b
Da hoa bDuy Duy
 
Da sinh b
Da sinh bDa sinh b
Da sinh bDuy Duy
 
Da toan b
Da toan bDa toan b
Da toan bDuy Duy
 
Các dạng câu hỏi và bài tập và giải chi ti&#787
Các dạng câu hỏi và bài tập và giải chi ti&#787Các dạng câu hỏi và bài tập và giải chi ti&#787
Các dạng câu hỏi và bài tập và giải chi ti&#787Duy Duy
 
Da hoaa ct_dh_12
Da hoaa ct_dh_12Da hoaa ct_dh_12
Da hoaa ct_dh_12Duy Duy
 
Da toan aa1
Da toan aa1Da toan aa1
Da toan aa1Duy Duy
 
Da ly aa1
Da ly aa1Da ly aa1
Da ly aa1Duy Duy
 
Da hoa a
Da hoa aDa hoa a
Da hoa aDuy Duy
 
Giaihhoathu1
Giaihhoathu1Giaihhoathu1
Giaihhoathu1Duy Duy
 
Giaidehoa1doc
Giaidehoa1docGiaidehoa1doc
Giaidehoa1docDuy Duy
 
Dethuhoa1
Dethuhoa1Dethuhoa1
Dethuhoa1Duy Duy
 
Lylan1doc
Lylan1docLylan1doc
Lylan1docDuy Duy
 

More from Duy Duy (20)

Bai tap a2 c2
Bai tap a2   c2Bai tap a2   c2
Bai tap a2 c2
 
Ngan hang a2 c2 ths. cao xuan phuong
Ngan hang a2 c2 ths. cao xuan phuongNgan hang a2 c2 ths. cao xuan phuong
Ngan hang a2 c2 ths. cao xuan phuong
 
Bt toan a2
Bt toan   a2Bt toan   a2
Bt toan a2
 
Bai tap a2 c2
Bai tap a2   c2Bai tap a2   c2
Bai tap a2 c2
 
Bt toan cao cap tap 1 nguyen thuy thanh
Bt toan cao cap tap 1 nguyen thuy thanhBt toan cao cap tap 1 nguyen thuy thanh
Bt toan cao cap tap 1 nguyen thuy thanh
 
A
AA
A
 
Da hoa b
Da hoa bDa hoa b
Da hoa b
 
Da sinh b
Da sinh bDa sinh b
Da sinh b
 
Da toan b
Da toan bDa toan b
Da toan b
 
Các dạng câu hỏi và bài tập và giải chi ti&#787
Các dạng câu hỏi và bài tập và giải chi ti&#787Các dạng câu hỏi và bài tập và giải chi ti&#787
Các dạng câu hỏi và bài tập và giải chi ti&#787
 
Da hoaa ct_dh_12
Da hoaa ct_dh_12Da hoaa ct_dh_12
Da hoaa ct_dh_12
 
Da toan aa1
Da toan aa1Da toan aa1
Da toan aa1
 
Da ly aa1
Da ly aa1Da ly aa1
Da ly aa1
 
Da hoa a
Da hoa aDa hoa a
Da hoa a
 
Hoalan2
Hoalan2Hoalan2
Hoalan2
 
Giaihhoathu1
Giaihhoathu1Giaihhoathu1
Giaihhoathu1
 
Giaidehoa1doc
Giaidehoa1docGiaidehoa1doc
Giaidehoa1doc
 
Dethuhoa1
Dethuhoa1Dethuhoa1
Dethuhoa1
 
Vatly
VatlyVatly
Vatly
 
Lylan1doc
Lylan1docLylan1doc
Lylan1doc
 

[Vnmath.com] 13-ki-thuat-giai-phuong-trinh-ham

  • 1. www.VNMATH.com PHƯƠNG TRÌNH HÀM - K THU T GI I VÀ M T S V N Đ LIÊN QUAN Tr n Minh Hi n - GV trư ng THPT chuyên Quang Trung, Bình Phư c Ngày 15 tháng 6 năm 2011 M cl c M cl c 1 1 Phương pháp th bi n 2 2 Phương trình hàm Cauchy 12 3 Phương pháp quy n p 19 4 Khai thác tính ch t đơn ánh, toàn ánh, song ánh, ch n l c a hàm s 24 5 Khai thác tính đơn đi u c a hàm s 34 6 Khai thác tính ch t đi m b t đ ng c a hàm s 40 7 Phương pháp đưa v phương trình sai phân 44 8 Phương pháp s d ng tính liên t c c a hàm s 46 9 ng d ng phương trình hàm cơ b n 53 10 B t đ ng th c hàm 60 11 Hàm tu n hoàn 65 12 M t s chuyên đ phương trình hàm 66 12.1 Phương trình hàm gi i nh tính giá tr hàm s theo hai cách khác nhau . . . . . . . . . . 66 13 Gi i phương trình hàm b ng cách thêm bi n 68 14 LUY N T P PHƯƠNG TRÌNH HÀM 69 14.1 Phương pháp th bi n . . . . . . . . . . . . . . . . . . . . . . . . . . . . . . . . . . . . . 69 14.2 B t đ ng th c hàm . . . . . . . . . . . . . . . . . . . . . . . . . . . . . . . . . . . . . . . 69 1
  • 2. www.VNMATH.com 1 PHƯƠNG PHÁP TH BI N 1 Phương pháp th bi n Phương pháp th bi n có l là phương pháp đư c s d ng nhi u nh t khi gi i phương trình hàm. Ta có th : • Ho c cho các bi n x, y, . . . nh n các giá tr b ng s . Thư ng các giá tr đ c bi t là 0, ±1, ±2, . . . • Ho c th các bi n b ng các bi u th c đ làm xu t hi n các h ng s ho c các bi u th c c n thi t. Ch ng h n, n u trong phương trình hàm có m t f (x + y) mà mu n có f (0) thì ta th y b i −x, mu n có f (x) thì cho y = 0, mu n có f (nx) thì th y b i (n − 1)x. Ví d 1.1. (Áo 199?) Tìm t t c các hàm s f : R → R th a mãn đi u ki n x2 f (x) + f (1 − x) = 2x − x4 , ∀x ∈ R. Gi i Thay x b i 1 − x ta đư c (1 − x)2 f (1 − x) + f (x) = 2(1 − x) − (1 − x)4 , ∀x ∈ R. Nhu v y ta có h 8 < x2 f (x) + f (1 − x) = 2x − x4 :f (x) + (1 − x)2 f (1 − x) = 2(1 − x) − (1 − x)4 . Ta có D = (x2 − x − 1) (x2 − x + 1) và Dx = (1 − x2 ) (x2 − x − 1) (x2 − x + 1). V y D.f (x) = Dx , ∀x ∈ R. T đó ta có nghi m c a bài toán là 8 > > < 1 − x2 : x = a, x = b, f (x) = >c ∈ R : x = a, (c là h ng s tùy ý), > : 2a − a4 − a2 c : x = b, v i a, b là nghi m c a phương trình x2 − x − 1 = 0. Nh n xét: Bài toán trên đư c dùng m t l n n a trong kỳ thi VMO 2000, b ng B. Ví d 1.2. Tìm t t c các hàm s f : R → R th a mãn đi u ki n f (x + y) + f (x − y) = 2f (x) cos y, ∀x, y ∈ R Hint: 1. Th y → π 2 2. Th y → y + π ho c th x = π 2 2 3. Th x → 0 Đáp s : f (x) = a cos x + b sin x(a, b ∈ R) Ví d 1.3. f : R → R th a mãn đi u ki n f (xy + x + y) = f (xy) + f (x) + f (y), x, y ∈ R. Ch ng minh r ng: f (x + y) = f (x) + f (y), ∀x, y ∈ R. GV: Tr n Minh Hi n . . . . . . PTH b i dư ng h c sinh gi i . . . . . . Trư ng THPT chuyên Quang Trung
  • 3. www.VNMATH.com 1 PHƯƠNG PHÁP TH BI N Hint: 1. Tính f (0) 2. Th y = −1, ch ng minh f là hàm l 3. Th y = 1 ⇒ f (2x + 1) = 2f (x) + 1 4. Tính f (2(u + v + uv) + 1) theo (3) và theo gi thi t đ suy ra f (2uv + u) = 2f (uv) + f (u) 5. 1 Cho v = − 2 , u → x và u → y, 2uv → x đ suy ra đi u ph i ch ng minh 2 Ví d 1.4. Tìm t t c các hàm s f : R → R đ ng th i th a mãn các đi u ki n sau: 1 f (x) = xf , ∀x = 0 x f (x) + f (y) = 1 + f (x + y), ∀x, y ∈ R, (x, y) = (0, 0); x + y = 0 Hint: 1. Tính f (0), f (−1) € Š € Š 2. Tính a + 1 v i a = f (1) = f x+1 = f x + 1 x+1 theo c hai đi u ki n. x+1 1 Đáp s : f (x) = x + 1 Nh n xét: Th thu t này áp d ng cho m t l p các bài toán g n tuy n tính 1 Ví d 1.5. Tìm t t c các hàm s f : R+ → R th a f (1) = 2 và ‚ Œ 3 3 f (xy) = f (x)f + f (y)f , ∀x, y ∈ R+ y x Hint: 1. Tính f (3) 3 2. Th y → x 1 Đáp s : f (x) = 2 Ví d 1.6. Tìm t t c các hàm s f : R∗ → R th a mãn đi u ki n: 1 f (x) + 2f = 3x, ∀x ∈ R∗ x 1 Hint: Th x → x 2 Đáp s : f (x) = x − x Ví d 1.7. Tìm t t c các hàm s f : R{0, 1} → R th a mãn đi u ki n: x−1 f (x) + f = 2x, ∀x, ∈ R{0, 1} x Hint: −1 Th x → x−1 , x → x−1 x 1 x−1 Đáp s : f (x) = x + 1−x − x Luy n t p: 2. Tìm t t c các hàm s f : Q+ → Q+ th a mãn đi u ki n: f (x + 1) = f (x) + 1, ∀x ∈ Q+ và f (x3 ) = f 3 (x), ∀x ∈ Q+ GV: Tr n Minh Hi n . . . . . . PTH b i dư ng h c sinh gi i . . . . . . Trư ng THPT chuyên Quang Trung
  • 4. www.VNMATH.com 1 PHƯƠNG PHÁP TH BI N Hint: 1. Quy n p f (x + n) = f (x) + n,‹ ∈ Q+ , ∀n ∈ N  ∀x 3 p p 2. V i q ∈ Q+ , tính f q + q2 theo hai cách. Đáp s : f (x) = x, ∀x ∈ Q+ Ví d 1.8. (VMO 2002). Hãy tìm t t c các hàm s f (x) xác đ nh trên t p s th c R và th a mãn h th c € Š f (y − f (x)) = f x2002 − y − 2001.y.f (x), ∀x, y ∈ R. (1) Gi i a) Th y = f (x) vào (1) ta đư c € Š f (0) = f x2002 − f (x) − 2002. (f (x))2 , ∀x ∈ R. (2) b) L i thay y = x2002 vào (1) thì € Š f x2002 − f (x) = f (0) − 2001.x2002 .f (x), ∀x ∈ R. (3) L y (2) c ng v i (3) ta đư c € Š f (x) f (x) + x2002 = 0, ∀x ∈ R. T đây suy ra v i m i giá tr x ∈ R thì ta có ho c là f (x) = 0 ho c là f (x) = −x2002 . Ta s ch ra r ng đ th a mãn yêu c u bài toán thì b t bu c ph i có đ ng nh t f (x) ≡ 0, ∀x ∈ R ho c f (x) ≡ −x2002 , ∀x ∈ R. Th t v y, vì f (0) = 0 trong c hai hàm s trên, nên không m t tính t ng quát ta có th gi s t n t i a = 0 sao cho f (a) = 0, và t n t i b 0 sao cho f (b) = −b2002 (vì ch c n thay x = 0 vào quan h (1) ta nh n đư c hàm f là hàm ch n). Khi đó th x = a và y = −b vào (1) ta đư c € Š f (−b) = f a2002 + b . V y ta nh n đư c dãy quan h sau 0 = −b2002 = f (b) = f (−b) € Š = f a2002 + b 0(mâu thu n vì 0 = 0) = 2002 2002 . − (a2002 + b) (mâu thu n vì − (a2002 + b) −b2002 ) B ng cách th l i quan h hàm ban đ u ta k t lu n ch có hàm s f (x) ≡ 0, ∀x ∈ R th a mãn yêu c u bài toán. Ví d 1.9. (Hàn Qu c 2003) Tìm t t c các hàm s f : R → R th a mãn f (x − f (y)) = f (x) + xf (y) + f (f (y)) , ∀x, y ∈ R. (4) GV: Tr n Minh Hi n . . . . . . PTH b i dư ng h c sinh gi i . . . . . . Trư ng THPT chuyên Quang Trung
  • 5. www.VNMATH.com 1 PHƯƠNG PHÁP TH BI N Gi i Nh n th y hàm f (x) ≡ 0 th a mãn yêu c u bài toán. Xét trư ng h p f (x) ≡ 0. a) Th x = f (y) vào (4) ta đư c x2 f (0) f (0) = 2f (x) + x2 → f (x) = − + , 2 2 hay f 2 (x) f (0) f (f (x)) = − + . 1 2 b) Th x = f (z), v i z là m t s thu c R thì ta đư c f (f (z) − f (y)) = f (f (z)) + f (z)f (y) + f (f (y)) . V i lưu ý là f 2 (y) f (0) f 2 (z) f (0) f (f (y)) = − + và f (f (z)) = − + , 2 2 2 2 thay vào quan h hàm trên ta đư c (f (z) − f (y))2 f (f (z) − f (y)) = − + f (0). (5) 2 c) Ti p theo ta ch ng t t p {f (x) − f (y)|x, y ∈ R} = R. Do f (x) ≡ 0 nên t n t i m t giá tr y0 sao cho f (y0 ) = a = 0. Khi đó t quan h (4) ta có f (x − a) = f (x) + xa + f (a) → f (x − a) − f (x) = ax + f a. Vì v ph i là hàm b c nh t c a X nên xa + f a có t p giá tr là toàn b R. Do đó hi u f (x − a) − f (x) cũng có t p giá tr là toàn b R, khi x ∈ R. Mà {f (x) − f (y)|x, y ∈ R} ⊃ {f (x − a) − f (x)|x ∈ R} = R, do đó {f (x) − f (y)|x, y ∈ R} = R. V y t quan h (5) ta thu đư c x2 f (x) = − + f (0), ∀x ∈ R. 2 M t khác ta l i có x2 f (x) = − + f (0), ∀x ∈ T (f ) 2 x2 nên f (0) = 0. Th l i th y hàm s f (x) = − , ∀x ∈ R th a mãn quan h hàm. 2 x2 K t lu n: Có hai hàm s th a mãn là f (x) = − , ∀x ∈ R ho c f (x) ≡ 0. 2 Nh n xét: Bài toán trên l y ý tư ng t bài thi IMO 1996: Tìm t t c các hàm s f : R → R th a mãn f (x − f (y)) = f (f (y)) + xf (y) + f (x) − 1, ∀x, y ∈ R. x2 Đáp s là f (x) = − + 1, ∀x ∈ R. 2 GV: Tr n Minh Hi n . . . . . . PTH b i dư ng h c sinh gi i . . . . . . Trư ng THPT chuyên Quang Trung
  • 6. www.VNMATH.com 1 PHƯƠNG PHÁP TH BI N Ví d 1.10. (Iran 1999) Xác đ nh các hàm s f : R → R th a mãn € Š f (f (x) + y) = f x2 − y + 4yf (x), ∀x, y ∈ R. Gi i a) Th y = x2 ta đư c € Š f f (x) + x2 = f (0) + 4x2 f (x), ∀x ∈ R. b) Th y = −f (x) ta đư c € Š f (0) = f f (x) + x2 − 4 (f (x))2 , ∀x ∈ R. C ng hai phương trình trên ta đư c € Š 4f (x) f (x) − x2 = 0, ∀x ∈ R. T đây ta th y v i m i x ∈ R thì ho c là f (x) ≡ 0 ho c là f (x) = −x2 . Ta ch ng minh n u hàm f th a mãn yêu c u bài toán thì f ph i đ ng nh t v i hai hàm s trên. Nh n th y f (0) = 0, t đó thay x = 0 ta đư c f (y) = f (−y), ∀y ∈ R, hay f là hàm ch n. Gi s t n t i a = 0, b = 0 sao cho f (a) = 0, f (b) = −b2 , khi đó thay x = a, y = −b ta đư c f (−b) = f (a2 + b) → f (b) = f (a2 + b). T đó ta có quan h sau 0 = −b2 = f (b) = f (−b) € Š = f a2 + b 0(mâu thu n vì 0 = 0) = 2 2 . − (a2 + b) (mâu thu n vì − (a2 + b) −b2 ) Do đó x y ra đi u mâu thu n. Th l i th y hàm s f (x) ≡ 0 th a mãn yêu c u. Nh n xét: 1. Rõ ràng bài toán VMO 2002 có ý tư ng gi ng bài toán này. 2. Ngoài phép th như trên thì bài toán này ta cũng có th th c hi n nh ng phép th khác như sau: 1€ 2 Š a) Th y = x − f (x) . 2 b) Th y = 0 đ có f (f (x)) = f (x2 ), sau đó th y = x2 − f (x). c) Th y = x − f (x) và sau đó là y = x2 − x. Ví d 1.11. Tìm hàm s f : R → R th a mãn đi u ki n: f (x − f (y)) = 2f (x) + x + f (y), ∀x, y ∈ R. (6) Gi i GV: Tr n Minh Hi n . . . . . . PTH b i dư ng h c sinh gi i . . . . . . Trư ng THPT chuyên Quang Trung
  • 7. www.VNMATH.com 1 PHƯƠNG PHÁP TH BI N Nh n th y hàm f (x) ≡ 0 không th a mãn yêu c u. Xét f (x) ≡ 0. a) Thay x b i f (y) vào (6) ta đư c f (0) f (f (y)) = −f (y) + . 2 b) L i thay x b i f (x) ta đư c f (f (x) − f (y)) = 2f (f (x)) + f (x) + f (y) ‚ Œ f (0) = 2 −f (x) + + f (x) + f (y) 2 = − (f (x) − f (y)) + f (0). Tuy nhiên vi c ch ng minh t p {f (x) − f (y)|x, y ∈ R} có t p giá tr là R chưa th c hi n đư c. c) T đây ta có f (f (x) − 2f (y)) = f ((f (x) − f (y)) − f (y)) = 2f (f (x) − f (y)) + f (x) − f (y) + f (y) = −2 (f (x) − f (y)) + 2f (0) + f (x) = − (f (x) − 2f (y)) + 2f (0). Ta s ch ng minh t p {f (x) − 2f (y)|x, y ∈ R} b ng v i R. Th t v y t n t i giá tr y0 ∈ R sao cho f (y0 ) = a = 0. Khi đó thay y = y0 vào (6) ta có f (x − a) − 2f (x) = x + a, ∀x ∈ R. Mà khi x ∈ R thì x + a có t p giá tr là R. Ch ng t t p {f (x − a) − f (x)|x ∈ R} = R. Mà {f (x) − 2f (y)|x, y ∈ R} ⊃ {f (x − a) − f (x)|x ∈ R} nên {f (x) − 2f (y)|x, y ∈ R} = R. Do đó t (c) ta k t lu n f (x) = −x + 2f (0), ∀x ∈ R. Thay vào (6) ta đư c f (0) = 0. K t lu n: Hàm s f (x) = −x, ∀x ∈ R th a mãn yêu c u bài toán. Ví d 1.12. (Belarus 1995) Tìm t t c các hàm s f : R → R th a mãn f (f (x + y)) = f (x + y) + f (x)f (y) − xy, ∀x, y ∈ R. Gi i Rõ ràng f khác h ng s . a) y = 0 vào đi u ki n bài toán ta đư c f (f (x)) = (1 + f (0)) f (x), ∀x ∈ R. b) Trong đ ng th c trên thay x b i x + y thì (1 + f (0)) f (x + y) = f (f (x + y)) = f (x + y) + f (x)f (y) − xy, đơn gi n ta đư c f (0).f (x + y) = f (x)f (y) − xy. (7) GV: Tr n Minh Hi n . . . . . . PTH b i dư ng h c sinh gi i . . . . . . Trư ng THPT chuyên Quang Trung
  • 8. www.VNMATH.com 1 PHƯƠNG PHÁP TH BI N c) Thay y = 1 vào (7) thì f (0)f (x + 1) = f (x)f (1) − x. d) L i thay y = −1 và x b i x + 1 vào (7) ta có f (0).f (x) = f (x + 1).f (−1) + x + 1. K t h p hai đ ng th c trên ta đư c € Š (f (0))2 − f (1)f (−1) f (x) = (f (0) − f (−1)) x + f (0). N u (f (0))2 − f (1)f (−1) = 0, thì thay x = 0 vào phương trình cu i cùng ta đư c f (0) = 0, nên theo (7) thì f (x)f (y) = xy. Khi đó f (x)f (1) = x, ∀x ∈ R, đi u này d n đ n (f (0))2 − f (1)f (−1) = −1, mâu thu n. V y (f (0))2 −f (1)f (−1) = 0, suy ra f (x) là m t đa th c b c nh t nên có d ng f (x) = ax+b. Thay vào quan h hàm ban đ u suy ra a = 1, b = 0. V y hàm s th a mãn yêu c u bài toán là f (x) = x, ∀x ∈ R. Nh n xét: N u ch u khó tính ta s tính đư c f (0) = 0 b ng cách th các bi n x, y b i hai s 0 và 1. Ví d 1.13. (VMO 2005) Hãy xác đ nh t t c các hàm s f : R → R th a mãn đi u ki n f (f (x − y)) = f (x)f (y) − f (x) + f (y) − xy, ∀x, y ∈ R. (8) Gi i a) Th x = y = 0 vào (8) ta đư c f (f (0)) = (f (0))2 . b) Th x = y vào (8) và s d ng k t qu trên thì (f (x))2 = (f (0))2 + x2 , ∀x ∈ R. Suy ra (f (x))2 = (f (−x))2 → |f (x)| = |f (−x)| , ∀x ∈ R. c) Th y = 0 vào (8) đư c f (f (x)) = f (0)f (x) − f (x) + f (0), ∀x ∈ R (∗). d) Th x = 0, y = −x vào (8) đư c f (f (x)) = f (0)f (−x) + f (−x) − a, ∀x ∈ R. T hai đ ng th c trên ta có f (0) (f (−x) − f (x)) + f (−x) + f (x) = 2f (0), ∀x ∈ R. (9) Gi s t n t i x0 = 0 sao cho f (x0 ) = f (−x0 ), thì th x = x0 vào (9) ta có f (x0 ) = f (0) → (f (x0 ))2 = (f (0))2 → (f (0))2 + x2 = (f (0))2 + 02 0 →x0 = 0 mâu thu n GV: Tr n Minh Hi n . . . . . . PTH b i dư ng h c sinh gi i . . . . . . Trư ng THPT chuyên Quang Trung
  • 9. www.VNMATH.com 1 PHƯƠNG PHÁP TH BI N V y f (x) = −f (x), ∀x ∈ R, t đi u này k t h p v i (9) ta có f (0) (f (x) − 1) = 0, ∀x ∈ R. T đây suy ra f (0) = 0, vì n u ngư c l i thì f (x) = 1, ∀x = 0, trái v i đi u ki n f là hàm l . T đây ta nh n đư c quan h quen thu c (f (x))2 = x2 , ∀x ∈ R. Gi s t n t i x0 ∈ R sao cho f (x0 ) = x0 , khi đó trong (*) ta có x0 = f (x0 ) = −f (f (x0 )) = −f (x0 ) = x0 , vô lý. V y ch ng t f (x) = −x, ∀x ∈ R. Th l i th y hàm này th a mãn bài toán. Nh n xét: Bài toán trên cho k t qu là hàm ch n f (x) = −x. N u v n gi a nguyên v ph i và đ nh n đư c hàm l f (x) = x, ta s a l i d ki n trong v trái như trong ví d sau Ví d 1.14. Tìm t t c các hàm s f : R → R th a mãn đi u ki n f (f (x) − y) = f (x) − f (y) + f (x)f (y) − xy, ∀x, y ∈ R. Gi i a) Th y = 0 ta đư c f (f (x)) = f (x) − f (0) + f (0).f (x), ∀x ∈ R. (10) b) Th y = f (x) và s d ng k t qu trên, ta đư c f (0) = f (x) − f (f (x)) + f (x).f (f (x)) − xf (x) (∗) = f (0) − 2f (0).f (x) + (f (x))2 + f (0). (f (x))2 − xf (x), hay −2f (0).f (x) + (f (x))2 + f (0). (f (x))2 − xf (x) = 0, ∀x ∈ R. c) Th x = 0 vào đ ng th c trên ta đư c (f (0))2 − (f (0))2 = 0 → f (0) = 0 ho c f (0) = 1. d) N u f (0) = 0 thì thay vào (10) ta có f (f (x)) = f (x), ∀x ∈ R, thay k t qu này vào trong (*) ta có f (x) = x. 1 e) N u f (0) = 1 thay vào (10) ta có f (f (x)) = 2f (x) − 1, thay vào trong (*) ta có f (x) = x + 1. 2 K t lu n: Thay vào ta th y ch có hàm s f (x) = x, ∀x ∈ R là th a mãn yêu c u. Ví d 1.15. (AMM,E2176). Tìm t t c các hàm s f : Q → Q th a mãn đi u ki n ‚ Œ x+y f (x) + f (y) f (2) = 2 và f = , ∀x = y. x−y f (x) − f (y) Gi i GV: Tr n Minh Hi n . . . . . . PTH b i dư ng h c sinh gi i . . . . . . Trư ng THPT chuyên Quang Trung
  • 10. www.VNMATH.com 1 PHƯƠNG PHÁP TH BI N Ta s ch ng minh f (x) = x là nghi m duy nh t c a bài toán d a vào m t chu i các s ki n sau. Trư c tiên nh n th y f không th là hàm h ng. a) Tính f (0), f (1). Thay y = 0 ta nh n đư c f (x) + f (0) f (1) = → (f (1) − 1) f (x) = f (0) (1 + f (1)) , ∀x ∈ Q. f (x) − f (0) Suy ra f (1) = 1, f (0) = 0. b) Hàm f là hàm l . Thay y = −x ta có 0 = f (0) = f (x) + f (−x) → f (−x) = −f (x), ∀x ∈ Q. c) Thay y = cx, c = 1, x = 0 ta có f (x) + f (cx) 1+c 1 + f (c) =f = , f (x) − f (cx) 1−c 1 − f (c) ‚ Œ p p f (p) suy ra f (cx) = f (c).f (x), l y c = q, x = thì ta đư c f = q q f (q) Ví d 1.16. Tìm t t c các hàm s f : R → R th a mãn € Š f (x − y)2 = (f (x))2 − 2xf (y) + y 2 , ∀x, y ∈ R. Gi i Thay x = y = 0 thì (f (0)) = (f (0))2 → f (0) = 0 ho c f (0) = 1. 1. N u f (0) = 0, thì thay x = y vào đi u ki n ban đ u ta đư c f (0) = (f (x))2 − 2xf (x) + x2 = (f (x) − x)2 → f (x) = x, ∀x ∈ R. Nh n th y hàm s này th a mãn. 2. N u f (0) = 1 thì l i v n thay x = y = 0 ta nh n đư c, v i m i x ∈ R thì ho c là f (x) = x + 1 ho c f (x) = x − 1. Gi s t n t i giá tr a sao cho f (a) = a − 1. Khi đó thay x = a, y = 0 ta đư c € Š f a2 = a2 − 4a + 1. Nhưng ta l i có ho c là f (a2 ) = a2 + 1 ho c là f (a2 ) = a2 − 1. Do đó ta ph i có ho c là 1 a2 − 4a + 1 = a2 + 1 ho c a2 − 4a + 1 = a2 − 1, t c a = 0 ho c là a = . Tuy nhiên ki m tra đ u 2 không th a. V y hàm s th a mãn yêu c u là f (x) = x, ∀x ∈ R ho c là f (x) = x + 1, ∀x ∈ R. Ví d 1.17. (THTT T9/361) Tìm t t c các hàm s f : R → R th a mãn đi u ki n € Š € Š f x3 − y + 2y 3 (f (x))2 + y 3 = f (x + f (y)) , ∀x, y ∈ R. GV: Tr n Minh Hi n . . . . . . PTH b i dư ng h c sinh gi i . . . . . . Trư ng THPT chuyên Quang Trung
  • 11. www.VNMATH.com 1 PHƯƠNG PHÁP TH BI N Gi i a) Thay y = x3 ta có € Š € Š f (0) + 2x3 3 (f (x))2 + x6 = f x3 + f (x) , ∀x ∈ R. b) Thay y = −f (x) ta đư c € Š € Š f x3 + f (x) − 2f (x) 3 (f (x))2 + (f (x))2 = f (0), ∀x ∈ R. T hai đ ng th c trên ta đư c € Š 2x3 3 (f (x))2 + x6 = 8 (f (x))3 , ∀x ∈ R. Do đó € Š 0 = 4 (f (x))2 − x3 3 (f (x))2 + x6 € Š € Š = 4 (f (x))3 − 4 (f (x))2 .x3 + (f (x))2 .x3 − x9 ‚ Œ2 ! € 3 Š€ 2 3 € 3 ŠŠ € 3 Š x3 15 = f (x) − x 4 (f (x)) + x f (x) + x = f (x) − x 2f (x) + + x6 . 4 16 ‚ Œ2 x3 15 Chú ý r ng 2f (x) + + x6 = 0 thì x = 0, f (0) = 0. B i v y trong m i trư ng h p ta đ u có 4 16 3 f (x) = x . Th l i th y hàm s này th a mãn bài toán. GV: Tr n Minh Hi n . . . . . . PTH b i dư ng h c sinh gi i . . . . . . Trư ng THPT chuyên Quang Trung
  • 12. www.VNMATH.com 2 PHƯƠNG TRÌNH HÀM CAUCHY 2 Phương trình hàm Cauchy PHƯƠNG TRÌNH HÀM CÔSI(HÀM TUY N TÍNH) Version 5.0 updated to 24 – 10 – 2008 I.Đ nh nghĩa: M t hàm s f : R → R g i là tuy n tính n u: f (x + y) = f (x) + f (y), ∀x, y ∈ R (Hàm s tuy n tính còn đư c g i là hàm Cauchy) II. M t s tính ch t Tính ch t 1. Hàm f tuy n tính và th a mãn x ≥ 0 th` f (x) ≥ 0, khi đóf là hàm đ ng bi n. (N u v i m i x ≥ 0 ⇒ f (x) ≤ 0 thì hàm ngh ch bi n). i Ch ng minh Xét x ≤ y ⇒ y − x ≥ 0 ⇒ f (y − x) ≥ 0 Ta có f (y) = f (y − x + x) = f (y − x) + f (x) ≥ f (x). V y f là hàm tăng. Tính ch t 2. Hàm tuy n tính f là hàm l . Ch ng minh Ta có f (0) = f (0 + 0) = 2f (0) ⇒ f (0) = 0. T đó f (0) = f (x + (−x)) = f (x) + f (−x) = 0 ⇒ f (−x) = −f (x), ∀x ∈ R. V y f là hàm l . Tính ch t 3. Hàm tuy n tính f liên t c t i x = 0 thì liên t c trên toàn t p s th c R. Ch ng minh Xét x0 ∈ R b t kỳ, ta có: lim [f (x) − f (x0 )] = lim [f (x) + f (−x0 )] = lim f (x − x0 ) = x→x0 x→x0 x→x0 lim f (y) = f (0) = f (0) = 0 V y hàm s liên t c t i x0 ∈ R. Do x0 l y b t kỳ trên R nên ch ng t hàm y→0 s liên t c trên toàn b R. Tính ch t 4. Hàm s f tuy n tính và đ ng bi n trên R thì liên t c trên R. Ch ng minh Cho y = 0 ⇒ f (x) = f (x) + f (0) ⇒ f (0) = 0 Cho y = x ⇒ f (2x) = 2f (x), b ng quy n p ta d dàng ch ng minh đư c: f (nx) = nf (x), ∀n ∈ N, ∀x ∈ € Š M t khác t công th c (1) suy € Š € Š R(1) ra f (x) = nf n hay f n = n f (x), ∀x ∈ R, ∀n ∈ N, do đó: f m x = m f (x), ∀x ∈ R, ∀m, n ∈ N x x 1 n n hay f (qx) = qf (x), ∀q ∈ Q, ∀x ∈ R Đ n đây ta có th gi i quy t theo hai cách sau: V i ε 0 b t ε kỳ, ch n δ = 1+|f (1)|+|f (−1)| , khi đó v i m i x ∈ R, |x| δ theo tính ch t c a t p s th c thì t n t i € Š m, n ∈ N sao cho |x| m δ, t c là − m x m . Vì f là hàm đ ng bi n nên f − m f (x) n n n n € Š f m ⇒ m f (−1) f (x) m f (1) ⇒ − m (1 + |f (1)| + |f (−1)|) f (x) m (1 + |f (1)| + |f (−1)|) V y n n n n n |f (x) − f (0)| = |f (x)| m (1 + |f (1)| + |f (−1)|) δ (1 + |f (1)| + |f (−1)|) = ε. Do đó hàm s liên n t c t i x = 0 nên liên t c trên R Ho c ta có th là như sau: t f (qx) = qf (x), ∀q ∈ Q, ∀x ∈ R nên f (x) = xf (1), ∀x ∈ Q Hơn n a v i m i x ∈ R, t n t i hai dãy h u t (un ), (vn ) ⊂ Q : un x vn mà lim un = lim vn = x. Do hàm đ ng bi n nên f (un ) f (x) f (vn ) ⇒ un f (1) f (x) vn f (1). n→∞ n→∞ Chuy n qua gi i h n ta đư c f (x) = f (1)x ∀x ∈ R hay f (x) = ax nên liên t c trên R. Tính ch t 5. Hàm tuy n tính f và liên t c trên R có bi u di n là f (x) = ax, (a = f (1)). Ch ng minh Theo cách thi t l p trong tính ch t 3 ta có f (x) = xf (1), ∀x ∈ Q. Vì v i m i x ∈ R, luôn t n t i dãy {xn }n∈N ⊂ Q sao cho lim xn = x. Vì f liên t c nên n→∞ lim f (xn ) = f (x) ⇒ lim xn f (1) = f (x) ⇒ f (x) = ax n→∞ n→∞ ,v i a = f (1) , th l i th y hàm s này th a mãn yêu c u bài toán. V y f (x) = ax, ∀x ∈ R. Tính ch t 6. Cho c 0. N u hàm s f tuy n tính và th a mãn đi u ki n |f (x)| ≤ c ∀x ∈ [−1, 1] thì f (x) = ax v i |a| ≤ c Ch ng minh T tính ch t 3 ta có f (qx) = qf (x), ∀q ∈ Q, x ∈ R Gi s (xn ) là dãy s th c = 0 th a mãn lim xn = 0. V i m i giá tr c a xn ta ch n m t s h u t qn th a mãn: √1 ≤ qn ≤ √1 , n = 1, 2, ...(có n→∞ 3 |xn | |xn | th t giá tr n = n0 , n0 +1, ... đ th a mãn đi u ki n trên) thì ta có: n→∞ qn = ∞ và n→∞ (xn .qn ) = 0 V y lim lim
  • 13.
  • 14. 1
  • 16. f .q .x
  • 17. qn n n = qn |f (qn xn )|, ∀n ∈ N, do n→∞ (xn .qn ) = 0 nên v i n đ l n thì qn xn ∈ [−1, 1] lim 1 nên |f (qn xn )| ≤ c, v i n đ l n. Do đó |f (xn )| ≤ qn c Do đó lim f (xn ) = 0 = f (0) nên hàm f liên t c n→∞ t i 0, t đó liên t c trên toàn b R do đó có bi u di n f (x) = ax. T đi u ki n bài toán ta đư c hàm c n tìm là f (x) = ax v i |a| ≤ c Tính ch t 7. N u hàm s f tuy n tính và th a mãn đi u ki n t n t i h ng s M 0 sao cho f (x) ≤ M ∀x ∈ [0, 1] thìf (x) = ax Ch ng minh T f (qx) = qf (x) ∀q ∈ Q, ∀x ∈ R GV: Tr n Minh Hi n . . . . . . PTH b i dư ng h c sinh gi i . . . . . . Trư ng THPT chuyên Quang Trung
  • 18. www.VNMATH.com 2 PHƯƠNG TRÌNH HÀM CAUCHY hay f (x) = ax ∀x ∈ Q T đi u ki n bài toán ta có: f (1) − f (x) = f (1 − x) ≤ M ∀x ∈ [0, 1], Suy ra f (1) − M ≤ f (x) ≤ M ∀x ∈ [0, 1] V y t n t i h ng s N 0 mà |f (x)| ≤ N ∀x ∈ [0, 1] ⇒ |f (x)| ≤ N ∀x ∈ [−1, 1](do f (−x) = −f (x)), đ n đây ta có th là ti p theo như tính ch t 6. đây
  • 20.
  • 21.
  • 22. minh khác như sau: V i m i x ∈ R, khi đó v i r ∈ Q+ sao cho |x| r thì
  • 23. x
  • 24. ≤ 1, do đó
  • 25. f x
  • 26. ≤ N .
  • 27.
  • 28. r
  • 29. r
  • 30.
  • 31. Š
  • 32. 1
  • 33. x
  • 34. r ∈ Q nên
  • 35. f r
  • 36. = 1 |f (x)| ≤ N ⇒ |f (x)| ≤ r.N Cho r → |x| thì |f (x)| ≤ N |x|. Suy ra lim f (x) = 0 r x→0 hay f liên t c t i 0 nên liên t c trên toàn b R. Do đó f (x) = ax Nh n xét 1. Cho t p A = R, [0, ∞) hay (0, ∞). N u f : A → R th a mãn f (x + y) = f (x) + f (y) và f (xy) = f (x)f (y), ∀x, y ∈ A, thì ho c là f (x) = 0, ∀x ∈ A ho c là f (x) = x, ∀x ∈ A Ch ng minh Theo tính ch t c a hàm c ng tính thì f (x) = f (1).x, ∀x ∈ Q. N u f (1) = 0 thì f (x) = f (x.1) = f (x).f (1) = 0, ∀x ∈ A. N u f (1) = 0 do T √ √ √ f (1) = f (1)f (1) ⇒ f (1) = 1 ⇒ f (x) = x, ∀x ∈ A Q N u y ≥ 0 thì f (y) = f ( y)f ( y) = f 2 ( y) ≥ 0 và do đó f (x + y) = f (x) + f (y) ≥ f (x), hay ch ng t f là hàm tăng. Bây gi v i m i x ∈ AQ, theo tính trù m t c a t p s th c, t n t i hai dãy pn , qn ∈ Q sao cho pn x qn ; pn x và qn x, khi n → ∞. Do f là hàm tăng, ta có: pn = f (pn ) ≤ f (x) ≤ f (qn ) = qn Chuy n qua gi i h n ta có f (x) = x, ∀x ∈ A III. Các h qu tr c ti p c a hàm Cauchy T quan h cho hàm f liên t c th a mãn đi u ki n f (x + y) = f (x) + f (y) ta có bi u di n c a hàm là f (x) = ax. N u ta đ t vào quan h hàm trên qua phép logarit Nepe t c là: ln f (x+y) = ln f (x)+ln f (y) = ln(f (x).f (y)), suy ra f (x+y) = f (x).f (y). V y n u f (x) 0 v i m i x ∈ R thì quan h hàm f (x + y) = f (x).f (y) d dàng chuy n v quan h hàm Cauchy qua phép logarit. Tuy nhiên t quan h hàm đó d dàng th y đư c bài toán v n gi i đư c v i mi n xác đ nh trên R. f (x) ≡ 0 H qu 1. Các hàm s liên t c trên R th a mãn đi u ki n: f (x+y) = f (x).f (y) (1) là: f (x) = ax (a 0) Ch ng minh Nh n th y hàm đ ng nh t f (x) ≡ 0 th a mãn quan h đó. Xét hàm không đ ng nh t 0, khi đó t n t i x0 :f (x0 ) = 0 thì: f (x0 ) = f ((x0 − x) + x) = Š (x0 − € Š (x) = 0 ⇒ f (x) = 0 ∀x ∈ R Và € f x)f x x 2 x cũng th a đi u ki n luôn dương, th t v y: f (x) = f 2 + 2 = f 2 0 ∀x ∈ R Do đó đ n đây ta ch c n đ t ln f (x) = g(x) thì ta có quan h : g(x + y) = g(x) + g(y) V y g(x) = bx, b ∈ R tùy ý. V y f (x) = ebx = ax (a 0). V y hai hàm th a mãn quan h đó là: B y gi l i t hàm Cauchy nêu ta nâng lũy th a c a bi n lên t x thành ex ta đư c quan h là f (e ) = f (ex ) + f (ey ) ⇒ g(x + y) = g(x) + g(y) v i g(x) = f (ex ) và hàm g thu đư c l i chính là hàm x+y Cauchy. M t khác t f (ex+y ) = f (ex ) + f (ey ) ⇒ f (ex .ey ) = f (ex ) + f (ey ), bây gi thay ngư c tr l i ex b i x thì ta đư c quan h m i là f (xy) = f (x) + f (y). Quan h này v i quan h Cauchy tương tác v i nhau b i vi c nâng lũy th a c a bi n. Tuy nhiên vi c nâng lũy th a c a bi n l i có yêu c u bi n ph i dương. N u có m t bi n b ng 0 thì bài toán tr nên d dàng v i k t qu là f (x) ≡ 0, n u c hai bi n cùng dương thì bài toán chuy n v phương trình hàm Cauchy qua phép nâng bi n lên lũy th a. N u c hai s cùng âm thì tích xy là s dương nên l i quy v trư ng h p hai bi n cùng dương. H qu 2. Các hàm s f (x)liên t c trên R{0} th a mãn đi u ki n:f (xy) = f (x) + f (y) ∀x, y ∈ R (2)là: f (x) = b ln |x| ∀x ∈ R{0}, b ∈ R Ch ng minh N u x = y = 1 thì t (3) ta đư c f (1) = 0. L i cho x = y = −1 ta đư c f (−1) = 0. Bây gi cho y = −1 thì ta đư c f (x) = f (−x) ∀x ∈ R. Do đó f là hàm ch n. a) Xét x, y ∈ R+ , đ t x = eu , y = ev , f (eu ) = g(u) ta đư c g(u + v) = g(u) + g(v) ∀u, v ∈ R ⇔ g(t) = bt ⇒ f (x) = a ln x ∀x ∈ R+ , a ∈ R b) N u x, y ∈ R− thì xy ∈ R+ nên v i y = x ta đư c: f (x) = 2 f (x2 ) = 1 b ln(x2 ) = b ln |x| ∀x ∈ R− , b ∈ R 1 2 L i ti p t c t quan h hàm f (x + y) = f (x).f (y) ta l i nâng bi n theo lũy th a c a e thì có d ng f (ex+y ) = f (ex )f (ey ) ⇒ f (ex .ey ) = f (ex )f (ey ) và ta đư c quan h hàm: g(xy) = g(x)g(y) Hi n nhiên bài toán có ngay l i gi i n u mi n xác đ nh ch a s 0. Do đó ta đ t v n đ đó như sau: H qu 3. Các hàm f (x) liên t c trên R{0} th a mãn đi u ki n:f (xy) = f (x)f (y), ∀x, y ∈ R{0}là: GV: Tr n Minh Hi n . . . . . . PTH b i dư ng h c sinh gi i . . . . . . Trư ng THPT chuyên Quang Trung
  • 37. www.VNMATH.com 2 PHƯƠNG TRÌNH HÀM CAUCHY 8 α xβ , ∀x ∈ R+ f (x) = 0 f (x) = |x| f (x) = Ch ng minh Thay y = 1 ⇒ f (x)(1 − f (1)) = : − |x|β , ∀x ∈ R− 0, ∀x ∈ R{0} (1)Š N u f (1) €=Š 1 thì t (1) suy ra f (x) ≡ 0, ∀x ∈ R{0} Xét f (1) = 1, khi đó € 1 1 1 = f (1) = f x. x = f (x)f x , ∀x ∈ R{0}. V y f (x) = 0, x ∈ R{0}. a) Xét x, y ∈ R+ , đ t x = eu , y = ev va g(t) = f (et ). Khi đó ta có: g(u + v) = g(u)g(v), ∀u, v ∈ R V yg(t) = at ∀t ∈ R(a 0 tuy y`) và do đó: f (x) = f (eu ) = g(u) = au = aln x = xln a = xα , ∀x ∈ R+ trong đóα = ln a b) u Bây gi ta xét trư ng h x = 0, y = 0 b t kỳ thì cho và x = y = −t ta nh n đư c f 2 (t) = f (t2 ) = p f (−t) = f (t) = tc (hay 0) f (−t)f (−t) = f 2 (−t) ⇒ V y trong trư ng h p t ng quát ta có các nghi m f (−t) = −f (t) = −tc 8 xβ , ∀x ∈ R+ là: a) f (x) = 0 b) f (x) = |x|α f (x) = : − |x|β , ∀x ∈ R− T quan h hàm Cauchy f (x + y) = f (x) + f (y) ta th c hi n v trái theo trung bình c ng v trái theo bi n và trung bình c ng v ph i theo hàm s thì ta nh n đư c: Š € H qu 4(Hàm Jensen). Các hàm f (x) liên t c trên R th a mãn f x+y = f (x)+f (y) (4) là:f (x) = ax+b € Š € Š 2 2 x f (x)+f (0) f (x)+f (y) x+y f (x+y)+f (0) Ch ng minh Cho y = 0 ⇒ f 2 = 2 . V y: 2 =f 2 = 2 ⇒ f (x + y) + f (0) = f (x) + f (y) Đ t g(x) = f (x) − f (0) thì ta có g(x + y) = g(x) + g(y) hay g(x) = ax⇒ f (x) = ax + b L i trong quan h hàm Jensen ta Š c hi n logarit Nepe n i t i c a bi n(dĩ nhiên trong trư ng € th √ h p các bi n dương, ta đư c:f ln x+ln y 2 = f (ln x)+f (ln y) ⇔ f (ln xy) = f (ln x)+f (ln y) . T v n đ này đ t 2 2 ngư c lŠ i ta đư c h qu sau: H qu 5. Các hàm f (x) xác đ nh và liên t c trên R+ th a mãn đi u ki n: €√ f xy = f (x)+f (y) ∀x, y ∈ R+ (5) là f (x) = a ln x + b Đi u ki n x, y ∈ R+ là đ cho hàm s luôn đư c 2 xác đ nh. Ch ng minh Đ t x = eu , y = ev , g(u) = f (eu ). Khi đó g(u) liên t c trên R và th a mãn đi u € Š ki n: g u+v = g(u)+g(v) ∀u, v ∈ R Suy ra g(u) = au + b ⇒ f (x) = a ln x + b, ∀x ∈ R+ . 2 2 € Š Cũng l i t quan h hàm f x+y = f (x)+f (y) n u ta vi t đư c vào dư i d ng c a bi u di n logarit 2 € Š € 2 Š È € Š È x+y ln f (x)+ln f (y) t c là: ln f 2 = 2 ⇒ ln f x+y = ln f (x)f (y) ⇒ f x+y = f (x)f (y) T c là ta 2 2 € Š È đư c quan h hàm: f x+y = f (x)f (y). V y ta có: H qu 6. Hàm s f : R → R liên t c th a 2 2 € Š È f (x) ≡ 0 f x+y = f (x)f (y) (6) là:4 2 f (x) = eax+b (a, b ∈ R) È Ch ng minh T đi u ki n bài toán cho x = y ⇒ f (x) = f 2 (x) ≥ 0. N u t n t i x0 : f (x0 ) = 0 thì: È € Š x0 +y f 2 = f (x0 )f (y) = 0 ∀y ∈ R t c là f (x) ≡ 0 N u f (x) 0 thì th c hi n logarit Nepe hai v đưa v hàm Jensen ta đư c:f (x) = eax+b , a, b tùy ý thu c R. T đó ta có đi u ph i ch ng minh. L i t quan h hàm trong h qu 5, th c hi n phép toán ngh ch đ o hàm s (gi s th c hi n 1 1 1 f (x) + f (y) 1 đư c) ta có: √ f ( xy) = 2 , b ng cách đ t g(x) = ta nh n đư c h qu sau: H qu 7. Các f (x) + √ 2 hàm f (x) xác đ nh và liên t c trên R th a mãn đi u ki n:f ( xy) = 1 + 1 ∀x, y ∈ R+ (7) là f (x) f (y) hàm h ng f (x) = b ∈ R{0} Ch ng minh T gi thi t bài toán suy ra f (x) = 0 ∀x ∈ R+ . Ta 1 + 1 √ có f (√xy) = f (x) 2 f (y) ⇒ g( xy) = g(x)+g(y) ∀x, y ∈ R+ v i g(x) = f (x) Theo h qu 5 thì g(x) = 1 2 1 a ln x + b ⇒ f (x) = a ln1 . Đ f (x) liên t c trên R+ thì: a ln x + b = 0, ∀x ∈ R+ nên a = 0, b = 0. V y x+b f (x) = b ∈ R{0}(đpcm). 1 1 1 f (x) + f (y) T quan h hàm Jensen n u ta th c hi n ngh ch đ o(v i hàm s ) thì ta có: = f( x+y = € Š ) 2 2 f (x)+f (y) 2f (x)f (y) hay f x+y = f (x)+f(y) Tuy nhiên đ đ m b o cho phép ngh ch đ o hàm luôn th c hi n đư c 2 2f (x)f (y) thì ta ch c n gi i h n giá tr hàm trong R+ . Do đó ta nh n đư c k t qu : H qu 8. Hàm s f : R → R+ GV: Tr n Minh Hi n . . . . . . PTH b i dư ng h c sinh gi i . . . . . . Trư ng THPT chuyên Quang Trung
  • 38. www.VNMATH.com 2 PHƯƠNG TRÌNH HÀM CAUCHY € Š liên t c th a mãn f x+y = f (x)+f(y) (8) là f (x) = 1 , b 0 2 2f (x)f (y) b 1 Ch ng minh Ch c n đ t g(x) = f (x) , ta nh n đư c quan h hàm Jensen theo hàm g(x) nêng(x) = 1 cx + d. Do đó f (x) = cx+d . Tuy nhiên hàm s này c n ph i th a mãn đi u ki n f (x) ∈ R+ nên: 1 cx+d 0, ∀x ∈ R ⇒c = 0, b 0, v y hàm thu đư c là f (x) = 1 , b 0 tùy ý. b L i v n trong quan h hàm Jensen n u ta th c hi n phép bình phương vào hàm s thì ta nh n ngay đư c h qu sau: q € Š 2 2 H qu 9. Hàm s f (x)liên t c trên R th a f x+y = [f (x)] +[f (y)] (9) là f (x) = c v i c ≥ 0. Ch ng 2 2 € € ŠŠ2 2 2 minh T quan h hàm s suy ra f (x) ≥ 0, ∀x ∈ R. Ta có: f x+y = [f (x)] +[f (y)] . Đ t g(x) = [f (x)]2 2 2 √ thì ta nh n đư c quan h hàm Jensen cho hàm g(x)nên g(x) = ax + b. Do đó f (x) = ax + b. Mà theo √ đi u ki n thì ax + b ≥ 0, ∀x ∈ R ⇒ a = 0, b ≥ 0 Ta đư c hàm f (x) = b, b ≥ 0. T quan h hàm trong h qu 6, n u ta th È hi n phép nâng lũy th a lên cơ s e(đ i v i bi n) thì c x+y È √ x )f (ey ) ⇒ f ( ex .ey ) = ta có: f e 2 = f (e f (ex )f (ey ) Thay ngư c l i bi n d ng bình thư ng ta nh n đư c k t qu : È √ H qu 10. Hàm s f (x) xác đ nh và liên t c trên R+ th a f ( xy) = f (x)f (y), ∀x, y ∈ R+ (10) là: f (x) ≡ 0 € Š a Ch ng minh Đ t x = eu , y = ev , f (eu ) = g(u) thì ta nh n đư c: g u+v = 2 f (x) = c.x , a ∈ R, c 0 2 È g(u) ≡ 0 f (x) ≡ 0 g(u)g(v), theo h qu 6 thì: au .Vy4 . Trong quan g(u) = e + b f (x) = ea ln x+b = c.xa , c 0, a ∈ R √ 2 2 h hàm c a h qu 5, n u ta th c hi n theo quan h hàm bình phương, t c là f 2 ( xy) = f (x)+f (y) , 2 th c hi n căn b c hai hai v ta đư c h qu 11. H qu 11. Hàm s f (x) xác đ nh và liên t c trên q √ 2 2 R+ th a f ( xy) = f (x)+f (y) , ∀x, y ∈ R+ (11) là f (x) ≡ c, c ≥ 0 Ch ng minh T gi thi t c a 2 hàm dŠ th y f (x) ≥ 0, ∀x ∈ R+ . Đ t x = eu , y = ev , [f (eu )]2 = g(u). Khi đó g(u) ≥ 0, và ta có: € g u+v = g(u)+g(v) , ∀u, v ∈ R V y g(u) = au + b. Đ g(u) ≥ 0, ∀u ∈ Rthì a = 0, b ≥ 0. Do đó 2 2 f (x) ≡ c, c ≥ 0. € Š € Š L i t quan h hàm Jensen f x+y = f (x)+f (y) , ta xét phép gán hàm f (x) = g x thì ta nh n đư c 2 2 1 1 g ( 1 )+g ( 1 ) 2 g ( 1 )+g ( 1 ) quan h hàm s : g (x+y)/2 = x 2 y ⇔ g x+y = x 2 y , thay ngư c tr l i bi n bình thư ng ta đư c: H qu 12. Hàm s f (x) liên t c trên R{0} th a mãn „ Ž 2 f (x) + f (y) f 1 1 = , ∀x, y, x + y = 0 x + y 2 a (12) là hàm s f (x) = x + b; a, b ∈ R tùy ý. Gi i V i cách thi t l p như trên thì ta có g(x) = ax + b, € Š € Š v i g(x) = f x , khi đó thì f (x) = x + b; a, b ∈ R. L i t quan h hàm Jensen f x+y = f (x)+f (y) , ta 1 a 2 2 xét phép gán hàm f (x) = g 11 thì ta nh n đư c quan h hàm: (x) 1 1 € Š € Š + g 1 +g 1 ‚ Œ 2g 1 g 1 1 1 g( x ) 1 g( y ) x y 2 x y 2  ‹ = = € Š ⇔g = € Š = 1 1 g 1 2 2g 1 g 1 x+y g 1 +g 1 1 g( x ) + 1 g( y ) x+y x y x y 2  ‹ 2 Thay ngư c l i bi n ta đư c: H qu 13. Hàm s f (x) xác đ nh liên t c trên R{0} th a f 1 + 1 = x y 2 x 6 f (x) = , a = 0 2 6 a 1 (13) là 4 . B ng cách th c hi n các phép toán khai căn, nâng lũy th a, logarit 1 f (x) + f (y) 1 f (x) = , b = 0 b GV: Tr n Minh Hi n . . . . . . PTH b i dư ng h c sinh gi i . . . . . . Trư ng THPT chuyên Quang Trung
  • 39. www.VNMATH.com 2 PHƯƠNG TRÌNH HÀM CAUCHY Nepe như trong các ph n trư c ta thu đư c các k t qu tương t sau: H 2 qu 14. Hàm s f (x) xác đ nh  ‹ 2 È f (x) ≡ 0 liên t c trên R{0} th a f 1 + 1 = f (x)f (y), ∀x, y, x + y = 0(14) là: 4 a H qu x y f (x) = e x +b , a, b ∈ R  ‹ q 2 [f (x)]2 +[f (y)]2 15. Hàm s f (x) xác đ nh liên t c trên R{0} th a f 1 1 +y = 2 , ∀x, y, x + y = 0 (15) là: √ x + x2 +y 2 f (x) ≡ c, c ≥ 0 tùy ý. H qu 16. Các hàm f (x) ≥ 0 xác đ nh liên t c trên R th a f 2 = q 2 [f (x)] +[f (y)] 2 √ , ∀x, y ∈ R+ (16) là: f (x) = ax2 + b v i a, b ≥ 0 tùy ý. H qu 17. Các hàm s f (x) xác 2 √ x2 +y 2 đ nh, li n t c trên R và th a f 2 = f (x)+f (y) , ∀x, y ∈ R (17) là: f (x) = ax2 + b; ∀a, b ∈ R H 2 √ È x2 +y 2 qu 18. Các hàm s f (x) xác đ nh, li n t c trên R th a f 2 = f (x)f (y), ∀x, y ∈ R (18) là: 2 √ f (x) ≡ 0 x2 +y 2 4 H qu 19. Các hàm s f (x) xác đ nh, li n t c trên R th a f = 2 2 f (x) = eax +b ; ∀a, b ∈ R 2 1 1 + 1 , ∀x, y ∈ R (19) là: f (x) = ax2 +b v i ab ≥ 0, b = 0 tùy ý. f (x) f (y) IV. Các bài t p v n d ng Bài toán 1. Tìm t t c các hàm f (x) liên t c trên R th a: f (x + y) = f (x)+f (y)+f (x)f (y) Gi i: T bài toán ta có: f (x+y)+1 = (f (x)+1)(f (y)+1) nên đ t g(x) = f (x)+1 thì ta có g(x+y) = g(x).g(y) ⇒ g(x) = ax v y f (x) = ax −1. Bài toán 2. Tìm t t c các hàm s f (x) liên t c trên R th a mãn đi u ki n:f (x)+f (y)−f (x+y) = xy, ∀x, y ∈ R Gi i Ta có th vi t l i phương trình 1 f (x) + f (y) − f (x + y) = [(x + y)2 − (x2 + y 2 )] hàm dư i d ng: 2 1 Đ t g(x) = f (x) + 2 x2 thì ta có 1 2 1 2 1 2 ⇔ f (x) + x + f (y) + y = f (x + y) + (x + y) 2 2 2 g(x) là hàm liên t c trên R th a mãn đi u ki n: g(x) + g(y) = g(x + y) V y g(x) = ax, ∀x ∈ R, a là m t 1 h ng s th c, nên f (x) = − 2 x2 + ax. Th l i th y hàm này th a mãn yêu c u bài toán. Bài toán 3. Cho a ∈ R, tìm t t c các hàm liên t c f : R → R sao cho: f (x − y) = f (x) − f (y) + axy, ∀x, y ∈ R Gi i Cho x = 1, y = 0 ⇒ f (1) = f (1) − f (0) nên f (0) = 0. L i cho x = y = 1 ⇒ f (0) = f (1) − f (1) + a ⇒ a = 0. V y v i a = 0 thì không t n t i hàm s . Ta vi t l i quan h hàm f (x − y) = f (x) − f (y), ∀x, y ∈ R T đây ta đư c: f (x) = f (x + y − y) = f (x + y) − f (y) ⇒ f (x + y) = f (x) + f (y), x, y ∈ R V y f (x) = ax, ∀x ∈ R Bài toán 4. Tìm t t c các hàm s f (x) xác đ nh liên t c trên R+ th a mãn đi u ki n:f x = f (x) − f (y) ∀x, y ∈ R+ Gi i Đ t x = t → x = ty thay vào ta có: f (t) = f (ty) − f (y) ⇒ y y f (ty) = f (t) + f (y). V y f (x) = a ln x ∀x ∈ R+ , a ∈ R. Bài toán 5. Cho a, b ∈ R{0}, tìm các hàm f (x) xác đ nh liên t c trên R và th a mãn đi u ki n: f (ax + by) = af (x) + bf (y) ∀x, y ∈ R(1) Gi i Cho x = y = 0 vào (1) ta đư c: f (0)(a + b − 1) = 0 N u a + b = 1 thì f (0) = 0. V y đi u ki n Cauchy đư c th a mãn, nên khi đó thì f (ax) = af (x) và f (bx) = bf (x), và ta có quan h f (ax + by) = f (ax) + f (by), ∀x, y ∈ R. V y f (x) = x. N u a + b = 1 thì nh n giá tr tùy ý, v y ta ph i đ t m t hàm m i đ đư c quan h Cauchy là g(x) = f (x) − f (0) thì g(0) = 0 và tương t như ph n trình bày trên ta có f (x) = cx + d V y: f (ax + by) = a + b = 1 ⇒ f (x) = cx, c ∈ R af (x) + bf (y) ∀x, y ∈ R là: Nh n xét: V i cách làm tương t a + b = 1 ⇒ f (x) = cx + d, c, d ∈ R cho quan h f (ax + by) = af (x) + bf (y) Bài toán 6. Xác đ nh các hàm s f liên t c trên R th a mãn đi u ki n:f (2x − y) = 2f (x) − f (y), ∀x, y ∈ R Gi i Đ t g(x) = f (x) − f (0) thì g(0) = 0, t phương trình trên ta thu đư c: g(2x − y) = 2g(x) − g(y), ∀x, y ∈ R Cho y = 0 ⇒ g(2x) = 2g(x) và cho x = € ⇒ g(−y)Š = −g(y). Thay vào trên ta đư c: g(2x − y) = g(2x) − g(y), ∀x, y ∈ R V y 0 y g(x+y) = g 2. x − 1. −1 = g(x)−g(−y) = g(x)+g(y), ∀x, y ∈ R. Do đó: g(x) = ax, x ∈ R, a là s th c 2 GV: Tr n Minh Hi n . . . . . . PTH b i dư ng h c sinh gi i . . . . . . Trư ng THPT chuyên Quang Trung
  • 40. www.VNMATH.com 2 PHƯƠNG TRÌNH HÀM CAUCHY tùy ý. V y f (x) = ax+b, th l i th y hàm này th a mãn yêu c u bài toán. Bài toán 8(Đ ngh IMO 1979). Ch ng minh r ng m i hàm f : R → R th a mãn đi u ki n: f (xy+x+y) = f (xy)+f (x)+f (y), ∀x, y ∈ R khi và ch khi f (x + y) = f (x) + f (y), ∀x, y ∈ R Gi i D th y n u f tuy n tính thì f th a mãn h th c đ u tiên. Gi s f (xy + x + y) = f (xy) + f (x) + f (y), ∀x, y ∈ R đ t y = u + v + uv ta đư c: f (x + u + v + xu + xv + uv + xuv) = f (x) + f (u + v + uv) + f (xu + xv + xuv) Hoán đ i vai trò c a x và u ta đư c: f (u + x + v + ux + uv + xv + uxv) = f (u) + f (x + v + xv) + f (ux + uv + uxv) So sánh hai đ ng th c trên ta đư c: f (x) + f (u + v + uv) + f (xu + xv + xuv)= f (u) + f (x + v + xv) + f (ux + uv + uxv) Hay f (uv) + f (xu + xv + xuv) = f (xv) + f (xu + uv + xuv) L y x = 1 ta có f (u) + 2f (uv) = f (u + 2uv), theo ví d 4 ta có đi u ph i ch ng minh. Bài toán 9. Tìm t t c các hàm s f (x) liên t c trên R th a mãn đi u ki n:f (x)f (y) − f (x + y) = sin x. sin y, ∀x, y ∈ R Gi i Thay y = 0 ta có f (x)[f (0) − 1] = 0 ⇒ f (0) = 1, vì d dàng nh n th y f (x) ≡ 0, ∀x ∈ R không là nghi m c a phương trình. Thay y = −x ta nh n đư c: f (x)f (−x) − f (0) = −sin2 x, €∀x ∈ R ⇒ f (x)f (−x) = 1 − sin2 x = € Š Š € Š cos2 x, ∀x ∈ R(1). Thay x = π vào (1) ta đư c nên: f π .f − π = 0 Ho c f π = 0 thay vào 2 2 2 € Š € Š € Š 2 hàm ta đư c: −f x + π = sin x ⇒ f x + π = − sin x → f (x) = − sin x − π = cos x, ∀x ∈ R 2 € Š €2 Š € 2Š Ho c f − π = 0 thay vào hàm ta đư c: f x − π = sin x ⇒ f (x) = sin x + π = cos x, ∀x ∈ R 2 2 2 D dàng ki m tra l i th y f (x) = cos x là hàm th a mãn yêu c u bài toán. Bài toán 10. Tìm t t c các hàm s f : R → R th a mãn f (x + y − xy) + f (xy) = f (x) + f (y) (1) v i m i x, y ∈ R. Gi i Ta ch ng minh n u f là hàm s th a mãn đi u ki n bài toán thì hàm s F (x) = f (x + 1) − f (x) s th a mãn đi u ki n hàm Cauchy F (u + v) = F (u) + F (v) v i m i (u, v) ∈ ∆ = {(u, v) : u + v 0ho c u = v = 0 ho c u + v ≤ −4} Th t v y, gi s f là hàm s th a mãn đi u ki n (1). Ta đ nh nghĩa hàm s f ∗ (x, y) b i: f ∗ (x, y) = f (x) + f (y) − f (xy) D th y r ng hàm f ∗ th a mãn phương trình hàm: f ∗ (xy, z) + ∗ (x, y) = f (x, yz) + f ∗ (y, z)(1) M t khác ta có f ∗ y) = f (x + y − xy)(2) Thay (2) vào (1) f ∗ (x, 1 1 1 ta đư c: f xy + y − x + f (x + y − xy) = f (1) + f y + y − 1 , v i m i x, y = 0 Đ t xy + y − x = u + 1 và x + y − xy = v + 1(3) ta nh n đư c: f (u + 1) + f (v + 1) = f (1) + f (u + v + 1), v i m i u, v th a 1 mãn đi u ki n trên. B ng vi c c ng hai đ ng th c c a (3) ta có y + y = u + v + 2, đ có nghi m y = 0 ch trong trư ng h p D = {(u + v + 2)2 − 4 = (u + v)(u + v + 4) ≥ 0}. Đi u ki n này x y ra khi và ch khi ho c là u + v 0 ho c u + v = 0 ho c u + v + 4 ≤ 0. B ng vi c ki m tra đi u ki n ta th y bài toán đư c th a. N u f là m t nghi m c a bài toán thì f ph i có d ng f (x) = F (x − 1) + f (1)(1) v i m i x, trong đó F th a mãn phương trình hàm Cauchy F (x + y) = F (x) + F (y) v i m i x, y. Ch ng minh Theo ch ng minh trên, thì f có d ng v i F th a mãn phương trình Cauchy v i m i (u, v) ∈ ∆. Ta s ch ng minh r ng F th a mãn phương trình Cauchy v i m i (u, v) b t kỳ. Gi s , khi đó t n t i m t s th c sao cho các đi m (x, u), (x + u, v), (x, u + v) n m trong ∆ v i vi c xác đ nh x là: c đ nh (u, v) ∈ ∆ thì t các b t đ ng th c x + u 0, x + u + v 0 ta tìm đư c đi u ki n c a x. Nhưng khi đó: F (u) = F (x + u) − F (x) F (v) = F (x + u + v) − F (x + u) Suy ra t các phương trình này ta có F (u) + F (v) = F (u + v). Và bài F (u + v) = F (x + u + v) − F (x) toán đư c ch ng minh. Bài toán 14(VMO 1992 b ng B). Cho hàm s f : R → R th a mãn f (x + 2xy) = f (x) + 2f (xy), ∀x, y ∈ R. Bi t f (1991) = a, hãy tính f (1992) Gi i Thay x = 0 ta đư c f (0) = 0. Thay y = −1 ta € Š nh n đư c f (x) = −f (−x). Thay y = − 2 ta đư c f (x) = 2f x . Xét x = 0 và s th c t b t kỳ, đ t 1 € Š 2 t t y = 2x ta nh n đư c: f (x + t) = f (x) + 2f 2 = f (x) + f (t) V y f là hàm Cauchy nên f (x) = kx, v i k là h ng s nào đó. T f (1991) = a ⇒ k.1991 = a ⇒ k = 1991 . Do đó f (1992) = 1992 a Bài toán a 1991 15. Tìm t t c các hàm s f (x) xác đ nh trên (0, +∞), có đ o hàm t i x = 1 và th a mãn đi u ki n √ √ f (xy) = xf (y) + yf (x), ∀x, y ∈ R+ Gi i Xét các hàm s sau g(x) = f√x . T gi thi t c a bài toán (x) √ √ √ ta có: xy.g(xy) = xy.g(x) + xy.g(y) ⇔ g(xy) = g(x) + g(y), ∀x, y ∈ R+ V y g(x) = loga x, x 0. GV: Tr n Minh Hi n . . . . . . PTH b i dư ng h c sinh gi i . . . . . . Trư ng THPT chuyên Quang Trung
  • 41. www.VNMATH.com 2 PHƯƠNG TRÌNH HÀM CAUCHY √ T đó ta có k t qu hàm s f (x) = k. x.loga x v i k ∈ R. L i t (1) n u ta đ t z = x + y thì y = z − x và quan h (1) tr thành f (z) = f (x).f (z − x), n u v i gi thi t f (x) = 0 ∀x ∈ R thì ta có th vi t l i f (z) như sau: f (z − x) = f (x) , và ta đ xu t đư c bài toán sau đây: Bài toán 18. Xác đ nh các hàm s f (x) 8 f (x − y) = f (x) , ∀x, y ∈ R liên t c trên R th a mãn đi u ki n: f (y) (2) Vì gi thi t là f (x) = 0 ∀x ∈ R : f (x) = 0 ∀x ∈ R x nên ch có hàm s f (x) = a (a 0) th a mãn yêu c u bài toán. To be continued . GV: Tr n Minh Hi n . . . . . . PTH b i dư ng h c sinh gi i . . . . . . Trư ng THPT chuyên Quang Trung
  • 42. www.VNMATH.com 3 PHƯƠNG PHÁP QUY N P 3 Phương pháp quy n p Phương pháp này yêu c u ta trư Šc h t tính f (0), f (1) r i d a vào đó tính f (n) v i n ∈ N. Sau đó € 1 tính f (n) v i n ∈ Z. Tính ti p f n , t đó suy ra bi u th c c a f (r) v i r ∈ Q. Phương pháp này thư ng s d ng khi c n tìm hàm s xác đ nh trên N, Z, Q. Ví d 3.1. Tìm t t c các hàm s f : Q → Q th a mãn đi u ki n: f (1) = 2, f (xy) = f (x)f (y) − f (x + y) + 1, ∀x, y ∈ Q. (11) Gi i Cho y = 1 và s d ng gi thi t f (1) = 2 ta đư c f (x + 1) = f (x) + 1, ∀x ∈ Q. (12) B ng phương pháp quy n p ta ch ng minh đư c f (x + m) = f (x) + m, ∀x ∈ Q, ∀m ∈ N. (13) Ti p theo ta s l n lư t ch ng minh: a) f (n) = n + 1, ∀n ∈ N. Th t v y trong (12) cho x = 0 ta tìm đư c f (0) = 1. Gi s ta đã có f (k) = k + 1 thì f (k + 1) = f (k) + 1 = k + 1 + 1 = k + 2. b) Ti p theo ta ch ng minh f (m) = m+1, ∀m ∈ Z. Th t v y, trong (12) cho x = −1 ta đư c f (−1) = 0. Trong (11) cho y = −1 thì ta có f (−x) = −f (x − 1) + 1, ∀x ∈ Q. Khi đó v i m ∈ Z, m 0 thì đ t n = −m, khi đó n ∈ N nên s d ng k t qu trên và ph n (a) ta đư c f (m) = f (−n) = −f (n − 1) + 1 = −n + 1 = m + 1. 1 c) Ti p theo ta ch ng minh f (x) = x + 1, ∀x ∈ Q. Trư c tiên ta tính f , n ∈ N+ , b ng cách trong n 1 (11) cho x = n, y = ta có n 1 1 2 = (n + 1)f −f n+ + 1. n n L i theo (13) thì 1 1 f n+ =f +n n n thay vào phương trình trên ta đư c 1 n+1 1 f = = + 1. n n n GV: Tr n Minh Hi n . . . . . . PTH b i dư ng h c sinh gi i . . . . . . Trư ng THPT chuyên Quang Trung
  • 43. www.VNMATH.com 3 PHƯƠNG PHÁP QUY N P m T đây thì v i x ∈ Q thì x luôn đư c bi u di n dư i d ng x = , m ∈ Z, n ∈ N+ , do đó n  m‹ f (x) = f n 1 = f m. n 1 1 = f (m).f −f m+ +1 n n 1 1 = (m + 1). +1 −f −m+1 n n 1 1 = (m + 1) +1 − −1−m+1 n n m = +1=x+1 n Th l i th y hàm s f (x) = x + 1, ∀x ∈ Q th a mãn yêu c u bài toán. Nh n xét: Bài toán trên k t qu không thay đ i n u ta làm trên t p R và không c n cho trư c f (1). Vi c cho trư c f (1) giúp quá trình quy n p thu n l i hơn. T l i gi i trên ch c n s lý trên t p s vô t . Tham kh o thêm v bài này trong bài 8.11. Ví d 3.2. Tìm t t c các hàm s liên t c f : R → R th a mãn f (x + y) + f (x − y) = 2 (f (x) + f (y)) , ∀x, y ∈ R. Gi i a) f (0) = 0, th t v y ch c n thay x = y = 0 ta có đư c k t qu . b) f là hàm ch n. Đ i vai trò gi a x, y trong đi u ki n ta có f (x + y) + f (y − x) = 2 (f (x) + f (y)) , ∀x, y ∈ R. Và như v y thì f (x − y) = f (y − x), ∀x, y ∈ R. Do đó f là hàm ch n nên ta ch c n làm vi c trên R+ . c) f (nx) = n2 f (x), ∀n ∈ N, ∀x ∈ R+ . Th t v y, cho x = y ta đư c f (2x) = 4f (x), ∀x ∈ R+ . Gi s ta đã có f (nx) = n2 f (x), ∀n ∈ N, ∀x ∈ R+ . Khi đó thay y = nx ta đư c f ((n + 1)x) + f (−(n − 1)x) = 2 (f (x) + f (nx)) , hay € Š f ((n + 1)x) = 2 f (x) + n2 f (x) − (n − 1)2 f (x) = (n + 1)2 f (x). d) f (qx) = q 2 f (x), ∀x ∈ R+ , ∀q ∈ Q+ . Th t v y t (c) thì  ‹ 1 x 1 f (x) = 2 f (nx) → f = 2 f (x), ∀n ∈ N, ∀x ∈ R+ . n n n m V i q ∈ Q+ thì q = v i m, n ∈ N, n = 0 nên n  x‹ 2  ‹ x m2 f (qx) = f m. =m f = 2 f (x) = q 2 f (x). n n n GV: Tr n Minh Hi n . . . . . . PTH b i dư ng h c sinh gi i . . . . . . Trư ng THPT chuyên Quang Trung
  • 44. www.VNMATH.com 3 PHƯƠNG PHÁP QUY N P e) Do f liên t c trên R+ nên f (x) = ax2 , ∀x ∈ R+ (v i a = f (1)). Th l i th y hàm s f (x) = ax2 , ∀x ∈ R th a mãn yêu c u bài toán. Nh n xét: Quan h bài toán trên chính là đ ng th c hình bình hành quen thu c. Đó là n u →, → − − u v là hai vector thì ta có |→ + →| + |→ − →| = 2 |→| + |→| − − 2 u v − − 2 u v − 2 u − 2 v B n ch t c a l i gi i là ch ng minh n u hàm f liên t c và th a mãn h ng đ ng th c hình bình hành thì b t bu c ph i có d ng f (x) = f (1)x2 . Cũng c n lưu ý là đi u ki n liên t c có th thay b ng đi u ki n đơn đi u c a hàm s . Ví d 3.3. Tìm t t c các hàm s f : [0, ∞) → R sao cho f đơn đi u và th a mãn đi u ki n € Š (f (x) + f (y))2 = f x2 − y 2 + f (2xy), ∀x ≥ y ≥ 0. Gi i 1 Cho x = y = 0 ta đư c f (0) = 0 ho c f (0) = . 2 1 1 1 a) Trư ng h p f (0) = , thì thay x = 1, y = 0 ta l i đư c f (1) = − ho c f (1) = . 2 2 2 1 1 (i) N u f (1) = − thì thay x = y = 1 ta đư c f (2) = . Khi đó ta th y f (0) f (1), f (1) f (2), 2 2 mâu thu n v i tính ch t đơn đi u c a hàm s . 1 (ii) V y f (1) = . Khi đó thay x = y ta đư c 2 € Š 1 4 (f (x))2 = f 2x2 + . 2 Xét dãy s x1 = 1, xn+1 = 2x2 , thay vào quan h trên ta đư c n 1 4 (f (xn ))2 = f (xn+1 ) + . 2 1 B ng quy n p ta đư c f (xn ) = v i m i n ∈ Z+ . Vì xn → ∞ và f đơn đi u nên suy ra 2 1 f (x) = v i m i x ≥ 0. 2 b) Trư ng h p f (0) = 0. Khi đó thay y = 0 ta đư c € Š f x2 = (f (x))2 , ∀x ≥ 0 → f (x) ≥ 0, ∀x ≥ 0. Ngoài ra thay x = y ta đư c 4 (f (x))2 = f (2x2 ). K t h p v i đ ng th c trên ta đư c 4f (x) = f (2x), ∀x ≥ 0. Trong phương trình hàm ban đ u, đ t x = u + v, y = u − v thì ta đư c € Š [f (u + v) − f (u − v)]2 = f (4uv) + f 2(u2 − v 2 ) ” — = 4 f (2uv) + f (u2 − v 2 ) = 4 (f (u) + f (v))2 . GV: Tr n Minh Hi n . . . . . . PTH b i dư ng h c sinh gi i . . . . . . Trư ng THPT chuyên Quang Trung
  • 45. www.VNMATH.com 3 PHƯƠNG PHÁP QUY N P T đây l y căn b c hai ta đư c f (u + v) + f (u − v) = 2 (f (u) + f (v)) , ∀u ≥ v ≥ 0. Phương trình hàm này có nghi m là f (x) = f (1)x2 , ∀x ≥ 0. Ngoài ra d dàng tính đư c f (1) = 0 ho c f (1) = 1. 1 K t lu n: Các hàm s th a mãn là f (x) ≡ 0, f (x) ≡ và f (x) = x2 , ∀x ≥ 0. 2 2 2 Nh n xét: Bài toán trên xu t phát t m t h ng đ ng th c quen thu c là (x2 + y 2 ) = (x2 − y 2 ) + (2xy)2 . Và đi m m u ch t c a bài toán là tính ch t f (x2 ) = (f (x))2 , đ suy ra f (x) ≥ 0 khi x ≥ 0. Ví d 3.4. (China 1996) Cho hàm s f : R → R th a mãn đi u ki n: f (x3 + y 3 ) = (x + y)(f 2 (x) − f (x)f (y) + f 2 (y)), ∀x, y ∈ R. Ch ng minh r ng f (1996x) = 1996f (x), ∀x ∈ R. Gi i a) Tính f (0) và thi t l p cho f (x). Cho x = y = 0 ta đư c f (0) = 0. Cho y = 0 ta đư c f (x3 ) = xf 2 (x). Nh n xét: f (x) và x luôn cùng d u. T đây ta có 1 1 f (x) = x 3 f 2 (x 3 ). b) Thi t l p t p h p t t c các giá tr a mà f (ax) = af (x). Đ t S = {a 0 : f (ax) = af (x), ∀x ∈ R}. • Rõ ràng 1 ∈ S. 1 • Ta ch ng t n u a ∈ S thì a 3 ∈ S. Th t v y 1 1 1 axf 2 (x) = af (x3 ) = f (ax3 ) = f (a 3 x)3 = a 3 x.f 2 (a 3 x) 2 1 ⇒ a 3 f 2 (x) = f 2 (a 3 x) 1 1 ⇒ a 3 f (x) = f (a 3 x) • N u a, b ∈ S thì a + b ∈ S. Th t v y 1 1 1 1 f ((a + b)x) = f (a 3 x 3 )3 + (b 3 x 3 )3 h i 1 1 1 1 1 1 1 1 1 1 = (a 3 + b 3 ) f 2 (a 3 x 3 ) − f (a 3 x 3 ).f (b 3 x 3 ) + f 2 (b 3 x 3 ) h i 1 1 2 1 1 2 1 1 = (a 3 + b 3 ) a 3 − a 3 b 3 + b 3 x 3 f 2 (x 3 ) = (a + b)f (x). B ng quy n p ta ch ng t m i n ∈ N đ u thu c S. Và bài toán ra là trư ng h p đ c bi t v i n = 1996. GV: Tr n Minh Hi n . . . . . . PTH b i dư ng h c sinh gi i . . . . . . Trư ng THPT chuyên Quang Trung
  • 46. www.VNMATH.com 3 PHƯƠNG PHÁP QUY N P Nh n xét: 1. N u ch đơn thu n ch ng minh k t qu c a bài toán thì có th quy n p tr c ti p. B ng cách kh o sát như trên ta s th y h t đư c t t c các giá tr c a a 0 mà f (ax) = af (x). 2. Do yêu c u “đ c bi t” c a bài toán, nên t nhiên ta s nghĩ ngay là có th ch ng minh đi u đó đúng v i m i s t nhiên, và qua đó, s nghĩ ngay đ n hư ng quy n p. 3. Vi c suy ra d u c a f (x) cùng d u v i x là quan tr ng, nó giúp ta tri t tiêu bình phương mà không c n xét d u, đây cũng là m t đi u đáng lưu ý trong r t nhi u bài t p khác. 4. Bài toán trên r t có th xu t phát t h ng đ ng th c x3 + y 3 = (x + y) (x2 − xy + y 2 ). Ví d 3.5. Tìm t t c các hàm f : Z → Z th a mãn: f (x3 + y 3 + z 3 ) = f 3 (x) + f 3 (y) + f 3 (z), ∀x, y, z ∈ Z Hint: 1. Tính f (0) và ch ng minh f là hàm l . 2. Ch ng t f (2) = 2f (1), f (3) = 3f (1). Ch ng minh b ng quy n p f (n) = nf (1), ∀n ∈ Z 3. Trong ch ng minh chuy n t n = k ≥ 0 sang n = k + 1, ta s d ng h ng đ ng th c sau: N u k ch n thì k = 2t, ta có: (2t + 1)3 + 53 + 13 = (2t − 1)3 + (t + 4)3 + (4 − t)3 khi k = 2t và n u k l thì k = 2t − 1 khi đó n = 2t luôn đư c vi t dư i d ng 2t = 2j (2i + 1), và đ ng th c trên ch c n nhân cho 23j Ví d 3.6. Tìm t t c các hàm f : N → N th a mãn các đi u ki n: f (1) 0 và f (m2 + n2 ) = f 2 (m) + f 2 (n), ∀m, n ∈ N Hint: 1. Tính f (0) ⇒ f (m2 + n2 ) = f (m2 ) + f (n2 ) 2. Ch ng minh f (n) = n, ∀n ≤ 10. V i n 10 ta s d ng các đ ng th c sau: (5k + 1)2 + 22 = (4k + 2)2 + (3k − 1)2 (5k + 2)2 + 12 = (4k + 1)2 + (3k + 2)2 (5k + 3)2 + 12 = (4k + 3)2 + (3k + 1)2 (5k + 4)2 + 22 = (4k + 2)2 + (3k + 4)2 (5k + 5)2 = (4k + 4)2 + (3k + 3)2 GV: Tr n Minh Hi n . . . . . . PTH b i dư ng h c sinh gi i . . . . . . Trư ng THPT chuyên Quang Trung
  • 47. www.VNMATH.com 4 KHAI THÁC TÍNH CH T ĐƠN ÁNH, TOÀN ÁNH, SONG ÁNH, CH N L C A HÀM S 4 Khai thác tính ch t đơn ánh, toàn ánh, song ánh, ch n l c a hàm s Trư c tiên ta nh c l i các khái ni m cơ b n này. a) N u f : R → R là đơn ánh thì t f (x) = f (y) ta suy ra đư c x = y. b) N u f : R → R là toàn ánh thì v i m i y ∈ R, t n t i x ∈ R đ f (x) = y. c) N u f : R → R là song ánh thì ta có c hai đ c trưng trên. N u m t hàm s mà đơn ánh chúng ta r t hay dùng th thu t tác đ ng f vào c hai v , n u m t hàm f toàn ánh ta hay dùng: T n t i m t s b sao cho f (b) = 0, sau đó tìm b. N u quan h hàm là hàm b c nh t c a bi n v ph i thì có th nghĩ t i hai quan h này. Ví d 4.1. Tìm t t c các hàm s f : Q → Q th a mãn f (f (x) + y) = x + f (y), ∀x, y ∈ Q. Gi i Nh n xét, hàm đ ng nh t 0 không th a mãn bài toán. Xét f (x) ≡ 0. a) f đơn ánh, th t v y, n u f (x1 ) = f (x2 ) thì f (f (x2 ) + y) = f (f (x2 ) + y) → x1 + f (y) = x2 + f (y) → x1 = x2 . b) f toàn ánh, th t v y, vì t n t i y0 sao cho f (y0 ) = 0. Do đó v ph i c a đi u ki n là m t hàm s b c nh t c a x nên có t p giá tr là Q. c) Tính f (0), cho x = y = 0 và s d ng tính đơn ánh ta đư c f (f (0)) = f (0) → f (0) = 0. T đó thay y = 0 ta đư c f (f (x)) = x, ∀x ∈ Q. d) Thay x b i f (x) và s d ng k t qu trên(và đi u này đúng cho v i m i x ∈ Q vì f là toán ánh) thì f (x + y) = f (x) + f (y), ∀x, y ∈ Q. T đây ta đư c f (x) = ax thay vào bài toán ta nh n f (x) ≡ x ho c f (x) ≡ −x trên Q. Nh n xét: N u yêu c u bài toán trên t p R thì c n thêm tính ch t đơn đi u ho c liên t c. C th , các b n có th gi i l i bài toán sau (THTT, 2010): Tìm t t c các hàm s liên t c f : R → R th a mãn đi u ki n f (x + f (y)) = 2y + f (x), ∀x, y ∈ R. Ví d 4.2. Tìm t t c các hàm s f : R → R th a mãn f (xf (y) + x) = xy + f (x), ∀x, y ∈ R. GV: Tr n Minh Hi n . . . . . . PTH b i dư ng h c sinh gi i . . . . . . Trư ng THPT chuyên Quang Trung
  • 48. www.VNMATH.com 4 KHAI THÁC TÍNH CH T ĐƠN ÁNH, TOÀN ÁNH, SONG ÁNH, CH N L C A HÀM S Gi i Thay x = 1 vào đi u ki n hàm ta đư c f (f (y) + 1) = y + f (1), ∀y ∈ R. T đây suy ra f là m t song ánh. L y x = 1, y = 0 ta đư c f (f (0) + 1) = f (1) → f (0) = 0 do f đơn ánh. f (x) Bây gi v i x = 0, đ t y = − thay vào đi u ki n hàm ta đư c x f (xf (y) + x = 0 = f (0)) → xf (y) = x do f đơn ánh, hay f (y) = −1, t c là ‚ Œ f (x) f − = f (y) = −1 = f (b), x v i b là m t s th c nào đó(do f là m t toàn ánh). V y f (x) = −bx, ∀x = 0. K t h p v i f (0) = 0 thì vi t g p thành f (x) = −bx, ∀x ∈ R. Thay vào đi u ki n hàm s ta có đư c hai hàm th a mãn là f (x) ≡ x và f (x) ≡ −x. Nh n xét: Bài toán này có th gi i b ng cách th bi n như sau mà không c n dùng đ n tính song ánh c a hàm s . Thay x = 1 ta đư c f (f (y) + 1) = y + f (1), ∀y ∈ R. Ví d 4.3. (Đ ngh IMO 1988) Xác đ nh hàm s f : N → N th a mãn đi u ki n sau: f (f (n) + f (m)) = m + n, ∀m, n ∈ N. (14) Gi i a) Trư c tiên ta ki m tra f đơn ánh. Th t v y gi s f (n) = f (m), khi đó f (2f (n)) = f (f (n) + f (n)) = 2n, và f (2f (n)) = f (f (m) + f (m)) = 2m. Do đó m = n, nên f đơn ánh. b) Ta tính f (f (n)) theo các bư c sau: cho m = n = 0 trong (14) thì ta đư c f (2f (0)) = 0, l i cho m = 2f (0) vào trong (14) thì ta đư c f (f (n)) = n + 2f (0). GV: Tr n Minh Hi n . . . . . . PTH b i dư ng h c sinh gi i . . . . . . Trư ng THPT chuyên Quang Trung
  • 49. www.VNMATH.com 4 KHAI THÁC TÍNH CH T ĐƠN ÁNH, TOÀN ÁNH, SONG ÁNH, CH N L C A HÀM S c) Tác đ ng f vào c hai v c a (14) và s d ng k t qu trên, ta đư c f (f (f (n) + f (m))) = f (n) + f (m) + 2f (0). Ngoài ra theo quan h đ bài thì f (f (f (n) + f (m))) = f (n + m). T đây ta có f (n + m) = f (n) + f (m) + 2f (0). Cho m = n = 0 thì f (0) = 0, do đó quan h trên tr thành hàm c ng tính. V y f (n) = an. Thay vào quan h bài toán ta đư c f (n) = n, ∀n ∈ N. - Nh n xét: Quan h đơn ánh c a bài toán này không c n thi t trong l i gi i. Và bài toán này có th ch ng minh b ng quy n p trên N. Cách 2. N u xét trên Z+ thì ta có th ch ng minh b ng quy n p f (x) = x, ∀x ∈ N. T c là, dùng phương pháp, ta ch ng minh không còn t n t i hàm s nào khác. Trư c tiên ta tính f (1). Gi s f (1) = t 1, đ t s = f (t − 1) 0. Nh n th y r ng n u f (m) = n thì f (2n) = f (f (m) + f (m)) = 2m. Như v y f (2t) = 2, f (2s) = 2t − 2. Nhưng khi đó thì 2s + 2t = f (f (2s) + f (2t)) = f (2t) = 2 → t 1, đi u này vô lý. V y f (1) = 1. Gi s ta có f (n) = n thì f (n + 1) = f (f (n) + f (1)) = n + 1. V y f (n) = n, ∀n ∈ Z+ . Ví d 4.4. (Balkan 2000) Tìm t t c các hàm s f : R → R th a mãn đi u ki n: f (xf (x) + f (y)) = (f (x))2 + y, ∀x, y ∈ R. (15) Gi i a) Ta tính f (f (y)) b ng cách cho x = 0 vào (15) ta đư c f (f (y)) = (f (0))2 + y, ∀y ∈ R. b) Ch ng t f đơn ánh. Th t v y n u f (y1 ) = f (y2 ) thì f (f (y1 )) = f (f (y2 )). T đây theo ph n (a) thì f 2 (0) + y1 = (f (0))2 + y2 ⇒ y1 = y2 . c) Ch ng t f toàn ánh vì v ph i c a (15) là m t hàm b c nh t c a y nên có t p giá tr b ng R. K t h p hai đi u trên ta thu đư c f là m t song ánh t R vào R. GV: Tr n Minh Hi n . . . . . . PTH b i dư ng h c sinh gi i . . . . . . Trư ng THPT chuyên Quang Trung